Re: sen36 novamente!

2000-01-06 Por tôpico Marcos Paulo

Tome um triângulo ABC com ângulo do vértice  = 36o. Trace a bissetriz BD. O
triângulo BDC é semelhante au triângulo ABC. Supondo AB = 1 encontre o
comprimento da base BC. Traça -se a altura AH e temos um triângulo retângulo
AHB cujos ângulos são 18o e 72o. Depois de encontrados o seno e cosseno de
18 (ou 72) encontra-se o seno de 36 facilmente.

- Original Message -
From: Fabricio Damasceno [EMAIL PROTECTED]
To: [EMAIL PROTECTED]
Sent: Wednesday, January 05, 2000 1:09 PM
Subject: sen36 novamente!


 Recentemente discutiamos sobre o sen36 e alguem afirmou que
 determinaria o sen36 facilmente usando um triangulo isósceles cujo
 angulo do vertice e 36.
  Eu nao consegui determinar com tal recurso; poderiam voces me ajudar!?!
  Abracos e um 2000 cheio de progressoes!


 MailBR - O e-mail do Brasil -- http://www.mailbr.com.br
 Faça já o seu. É gratuito!!!




Um problema legal!

2000-03-18 Por tôpico Marcos Paulo

Para descobrir quantos degraus ficavam a mostra em uma escada rolante, duas
pessoas subiram esta escada contando seus degraus. A primeira pessoa subiu
um degrau de cada vez e contou 21 degraus, a segunda subiu dois degraus de
cada vez e contou 28 degraus. Quantos degraus fica a mostra nesta escada
rolante?

[]'s M.P.

 winmail.dat


RES: Novo regulamento

2000-03-18 Por tôpico Marcos Paulo


Quando nós receberemos as provas(mês)??
Obrigado!

-Mensagem original-
De: [EMAIL PROTECTED] [mailto:[EMAIL PROTECTED]]Em
nome de Nicolau C. Saldanha
Enviada em: sábado, 18 de março de 2000 19:02
Para: [EMAIL PROTECTED]; [EMAIL PROTECTED]
Assunto: Novo regulamento



Já é oficial e já foi enviado para as escolas o novo regulamento da OBM.
Ele também está em

http://www.obm.org.br/regulamento.htm
http://www.mat.puc-rio.br/~nicolau/olimp/regulamento.txt

Para quem estiver interessado em comparar, o regulamento antigo
está em

http://www.mat.puc-rio.br/~nicolau/olimp/regulamentoantigo.txt

Alguns aspectos do novo regulamento que devem ser salientados:

...

3) OS NÍVEIS

A OBM será realizada anualmente em três níveis, de acordo
com a escolaridade do aluno:

Nível 1 - para alunos matriculados na 5ª ou 6ª séries
do ensino fundamental quando da realização da 1a fase da OBM.

Nível 2 - para alunos matriculados na 7ª ou 8ª séries
do ensino fundamental quando da realização da 1a fase da OBM.

Nível 3 - para alunos matriculados em qualquer série
do ensino médio quando da realização da 1a fase da OBM
ou que, tendo concluído o ensino médio menos de um ano antes,
não tenham ingressado em curso de nível superior até a data
de realização da 1a fase da OBM.

...

8) A REALIZAÇÃO DA OBM

a) A primeira fase da OBM será realizada em todos os
colégios cadastrados (veja anexo 1). A responsabilidade do
recebimento da prova, impressão, aplicação, correção e
transmissão dos dados ao coordenador regional será de um
professor do colégio, denominado representante da OBM no
colégio, e que estará em contato permanente com o
coordenador de sua região (veja anexo 2).

Alunos podem pedir ao coordenador regional para fazer a
primeira fase sob sua responsabilidade direta se por qualquer
razão não for possível fazer a prova na escola. Cabe ao
coordenador regional analisar se tal pedido é viável e
procedente.

b) A segunda fase da OBM será realizada nos colégios que
tiverem maior número de estudantes promovidos. Caberá ao
coordenador regional estabelecer os locais onde esta fase
será realizada, distribuir todos os alunos classificados
nesses locais, solicitar a ajuda de todos os professores
representantes da OBM para a aplicação e correção das
provas, e transmissão dos resultados para a Secretaria da
Olimpíada (veja anexos 2 e 3). O coordenador regional
pode inclusive autorizar todas as escolas a aplicarem a
segunda fase.

Como na primeira fase, alunos podem pedir ao coordenador
regional para fazer a segunda fase sob sua responsabilidade
direta.

Devem ser enviadas cópias de todas as provas aos coordenadores
regionais; os coordenadores regionais devem rever a correção.

...

9) CRITÉRIOS DE INSCRIÇÃO E PROMOÇÃO

a)  Para a primeira fase, cada colégio poderá inscrever
todos os estudantes interessados.
Não existe nenhuma limitação no número de alunos por escola
e todo aluno que desejar participar deve participar.
A existência da OBM deve ser amplamente divulgada na escola.

...

[]s, Nicolau




Qdo chegam as provas???

2000-04-05 Por tôpico Marcos Paulo


1)Quando as provas da Olimpíada de Maio chegam às nossas mãos nos
colegios???
Já foram enviadas???

2) Sobre a olimpitada Brasileira: Tenho alunos que desistiram do segundo
grau para fazer a prova do Colegio Naval. Estes alunos, embora tenham 16 ou
17 e ate 18 anos, não fazem o segundo grau. Em que nivel posso inscrevê-los
para fazer a prova? (se posso)

[]' M.P.




RES: divisibilidadeXfatorial

2000-04-19 Por tôpico Marcos Paulo

Tem um metodo meio cansativo de fatorar o fatorial de um número n, consiste
em ir dividindo n por 2, o quociente por 2 o novo quaciente por 2 e assim
sucessivamente. o expoente do 2 será a soma dos quocientes obtidos. Dái
passa -se para o 3 e repete o processo para todos os primos menores q n.
No caso da pergunta basta notar que para se formar um 10 eh necessario um
fator 2 e um fator 5. como o fator 2 deve ter um expoente muito maior q o
fator 5, basta saber o expoente do 5. daí temos:
100:5 = 20 (logo existem 20 fatores 5)
20:5 = 4 (logo existem 4 fatores de 5^2)
4 :5 =0 (logo não existem fatores 5 ao cubo)
O expoente do 5 será 20 + 4
Disso concluimos q 100! eé divisível por 10^24 e portanto tb é por 10^11.
espero ter ajudado!
[] M.P.

-Mensagem original-
De: [EMAIL PROTECTED] [mailto:[EMAIL PROTECTED]]Em
nome de Mira
Enviada em: quarta-feira, 19 de abril de 2000 16:18
Para: [EMAIL PROTECTED]
Assunto: divisibilidadeXfatorial


Ola lista!

Como provar que 1x2x3x...x100 eh divisivel por 10^11 sem ficar "procurando"
os 10s? Existe alguma propriedade que possa usar?

Obrigado!

Mira






RES: Problema de inteiros

2000-04-23 Por tôpico Marcos Paulo

Tem um teorema (acho q eh do Euclides O Zé Paulo me corrija, pois foi meu
professor de Os Números equivalente a álgebra 1)que diz que o m.d.c. entre
dois números sempre pode ser escrito como uma combinação linear entre esses
números.
No seu problema temos m.d.c(a, b)= 1 = xa + yb = 1. (eh fácil chegar aos
possiveis valores de x e y pelo algoritmo de Euclides)daí basta multiplicar
toda a expressão pelo valor de n desejado. Equações do tipo xa + yb = n são
chamadas Equações Diofantinas, que são usadas pra resolver, por exemplo o
problema Chinês do resto (muito interessante por sinal)
Espero ter ajudado.
[]'s M.P.

-Mensagem original-
De: [EMAIL PROTECTED] [mailto:[EMAIL PROTECTED]]Em
nome de Ecass Dodebel
Enviada em: sábado, 22 de abril de 2000 17:42
Para: [EMAIL PROTECTED]
Assunto: Problema de inteiros


E ai, pessoal?

Eu estava tentando resolver um dos problemas propostos na última Eureka! e
acabei chegando em uma parte que consigo seguir adiante mas é muito
trabalhosa a minha prova, e não sei se está bem certa. Lá vai.

1) Sejam x e y dois números primos entre si. Provar que podemos obter
qualquer número somando múltiplos de x e de y.

Solução.
Queremos provar que para todo o x,y,n dados, podemos achar f e g de modo que

fx + gy = n  ( a soma de múltiplos de x e de y dão o n )

Isola-se o f, ou o g... no caso isolei o f:

f = (n - gy)/x

Agora nos basta encontrar g de modo que x | n - gy. Para quem sabe um
pouquinho de Teoria dos Números, eu acho que se variarmos o y num s.c.r.
então o n - gy será um s.c.r. módulo x, e estaria provado. Mas vamos por
partes:

Suponhamos que

n - g1y =/= n - g2y (mod x)'=/= incongruente
g1y =/= g2y (mod x)  == afirmação similar a x não divide y(g1-g2)

Como  mdc(x,y)=1 então

g1 =/= g2 (mod x)

Vale tambem que se g1 =/= g2 (mod x) então n - g1y =/= n - g2y (mod x).
Agora escolhemos x números incongruentes módulo x (g1,...,gx), ou seja, que
nunca deixem o mesmo resto na divisão por x. E necessariamente:

n - giy =/= n - gjy (mod x) para todo o i e j

Ou seja, nesses x números (n-g1y,...,n-gxy), todos são incongruentes módulo
x, e como existem apenas x restos possíveis na divisão por x,
necessariamente algum deles deixará resto zero na divisão por x, e portanto
haverá um g, tal que:

f = (n - gy)/x será inteiro, e está provado o enunciado.

2) Sejam x e y dois números primos entre si. Prove que existe um N, de modo
que para todo o n  N, podemos escolher múltiplos positivos de x e de y que
somados dão n. Nessas condições teremos que ter

Solução.
O problema pede para que mostremos que existem f e g positivos de modo que,
para n  N

fx + gy = n  (lembrando que é todo mundo inteiro nesse e-mail)

A minha idéia é a seguinte, claramente xy - yx = 0, e portanto para todo o a
vale axy - ayx = 0, daí:

fx + gy + axy - ayx = n
(f + ay)x + (g - ax)y = n, para qualquer a que escolhermos

Quero mostrar que existirá um a, a partir de um dado n, para que f + ay e g
- ax sejam ambos positivos.

Conseguimos escolher a de modo que (f + ay)x - (g - ax)y = fx - gy + 2ayx
esteja entre -yx e yx, basta mostrar que nesse intervalo teremos f+ay e g-ax
sempre positivos.

Tanto f+ay quanto g-ax podem ficar entre [ n-xy ; n+xy ], ou seja basta que
n-xy0 e portanto que n  xy. Logo para N = xy vale o enunciado.


Obrigado para quem leu! E tem algum erro?
Valeu...

Get Your Private, Free E-mail from MSN Hotmail at http://www.hotmail.com





No Subject

2000-04-24 Por tôpico Marcos Paulo

Oi, já faz algum tempo que enviei uma mensagem perguntando sobre quando
receberemos a PROVA da Olimpíada de Maio. Ninguém respondeu!!! o q houve??
não devo fazer esse tipo de pergunta??? Se é proibido desculpem! É
ansiedade! A escola em que trabalho nunca participou de Olimpíadas e os
alunos cobram muito!
Também perguntei sobre alunos que já têm 17 ou 18 anos porém não estão
matriculados no segundo grau (para a OBM). O q respondo pra eles?? q eles
não podem participar?? ou q devem participar no nivel de segundo grau??

[]' M.P.

 winmail.dat


Um bom problema de geometria!

2000-05-01 Por tôpico Marcos Paulo

Dado um triângulo ABC tal que âng(A) = 60o, âng(B) = 50o, traça-se BD e CE
tais que âng(DBA) =10o e âng(ECA)=20o. Calcular âng(FED) onde F é a
interseção de BD com CE.

Minha solução (procuro outra):

Seja BF = a, e temos: EF = a*tg (10o); CF = a*tg (40o) e ainda FD = a*tg
(20o)*tg (40o).Ainda temos que tg(FED) =FD/EF e portanto temos tg(FED) =
a*tg(20o)*tg(40o)/a*tg(100) ou ainda tg(FED) = tg(40o)*tg(200) *tg(80o).
Usando uma identidade trigonométrica conhecida (a saber: tg(60o - x) * tg(x)
* tg (60o + x) = tg (3*x) ) vemos que tg (FED) = tg(3*20o) e portanto
âng(FED) = 60o.

Encontrei uma outra solução (sem o uso da trigonometria) MUITO
trabalhosa que consiste em reconhecer um outro problema que se encaixa
perfeitamente na figura (na verdade em parte da figura) e provar uma
congruencia de triângulos formados depois de traçar linhas que para qualquer
aluno pareceria simplesmente mágica! Conto com os colegas da lista para
encontrar uma solução possível de se ensinar para uma turma de oitava serie.
[]'s M.P.

 winmail.dat


Re: Pergunta

2000-05-08 Por tôpico Marcos Paulo

Não é difícil perceber que o quadrilátero APQB será inscritível quando o
angulo BAM for igual ao angulo DQB.BAM =[(arco MD) + (arco DB)]/2 e DQB
=[(arco MC) +(arco DB)]/2.para que eles sejam iguais devemos ter arco CM =
arco MD , ou seja M é médio do arco CD.
[]'s M.P.




RES: Polinômios

2000-05-09 Por tôpico Marcos Paulo



Acho 
que a reazão entre os coeficientes deve ser constante, ou seja, P tem que ser 
idêntico a k*Q

  -Mensagem original-De: [EMAIL PROTECTED] 
  [mailto:[EMAIL PROTECTED]]Em nome de Luiz LeitãoEnviada 
  em: terça-feira, 9 de maio de 2000 00:57Para: LISTA DE 
  DISCUSSÃO DA OBMAssunto: Polinômios
  Uma questão interessante que eu vi em um livro 
  'e a seguinte, dado dois polinômios P(x) e Q(X), de mesmo grau, sendo P(x) 
  diferente de Q(x). Que relação os polinômios P(x) e Q(x) devem ter para que o 
  quociente P(x)/Q(x) independa de x, considerando m o grau dos dois polinômios 
  e que todos os coeficientes são números reais e que P(x) é um polinômio não 
  nulo.
   
  Obrigado


Re: teorema do binômio

2000-06-14 Por tôpico Marcos Paulo

Este teorema é chamado TEOREMA DA COLUNA!!!
assim que tiver mais tempo demnstro pra vc!
[]'s MP
- Original Message -
From: Marcelo Souza [EMAIL PROTECTED]
To: [EMAIL PROTECTED]
Sent: Wednesday, June 14, 2000 3:51 PM
Subject: teorema do binômio


 Olá
  Olhei a resolução de um problema e não consegui entender uma coisa. A
 resolução do problema deixa que

 (n+1)  ( 2 )   ( 3 ) ( n )
 (   )= (   ) + (   ) + ... + (   )
 ( 3 )  ( 2 )   ( 2 ) ( 2 )

 Eu não entendi muito bem. Se for isso mesmo alguém poderia demontrar?

 Obrigado
 Abraços
 Marcelo
 OBS:
 ( n )
 (   )= n!/k!(n-k)!
 ( k )


 
 Get Your Private, Free E-mail from MSN Hotmail at http://www.hotmail.com






Re: Permutação Caótica

2000-08-16 Por tôpico Marcos Paulo




  
  Como se calcula o número de permutações caóticas 
  em um conjunto com elementos repetidos?
  
  Uma pergunta equivalente a essa que talvez fique 
  mais clara seria ( se entendi bem) quantos são os anagramas da palavra 
  MATEMATICA em que nenhuma das letras ocupa a posicão ocupada na palavra 
  MATEMATICA
  acho que eh isso ... 
  quanto a solução 
  
  []'s MP


Re: Dia da semana

2000-08-19 Por tôpico Marcos Paulo

Com certeza o conceito de congruencia mod 7 será usado na resolução desse
problema. No entando há outras coisas que deve-se levar em consideraço. POr
exemplo o ano 2000 é um ano bissexto!
Se quero saber que dia da semana cairá o dia 19/08/2001, é simples, pois a
diferença em dias (de hj até a data) é de 365 == 1 (mod7) logo essa data
será em um domingo (visto que hj é sabado)
Se tivesse feito a mesma pergunta no dia 12/02/2000 (que tambám caiu em um
sabado) para o dia 12/02/2001 a diferença não seria 365 mas sim 366 ==2
(mod7) logo essa data deve ser em uma segunda feira!
Um algoritmo para prever qualquer data deveria levar em consideração essas
variações...
Espero ter ajudado
[]'s MP
- Original Message -
From: "Marcos Eike Tinen dos Santos" [EMAIL PROTECTED]
To: [EMAIL PROTECTED]
Sent: Saturday, August 19, 2000 12:38 AM
Subject: Re: Dia da semana


 Para isso usa-se congruência.

 a==b(mod n)

 Veja que é um modo simplificado de expressar que ao dividir tanto a quanto
b
 por n dará um resto único r.

 Logo: a==x(mod 7)

 Acredito que seja isso, pois a diferença são 7 dias.

 Ats,
 Marcos Eike




 -Mensagem Original-
 De: Wellington Ribeiro de Assis [EMAIL PROTECTED]
 Para: discusspio de problemas [EMAIL PROTECTED]
 Enviada em: Sexta-feira, 18 de Agosto de 2000 22:59
 Assunto: Dia da semana


  Prezados amigos
 
  Alguem sabe dizer como eh o algoritmo usado para se descobrir que dia
  da semana cai uma determinada data de um ano qualquer?
 
  Bons estudos e abraco a todos,
  Wellington





Olimpíada Estadual do Rio de Janeiro

2000-08-23 Por tôpico Marcos Paulo



Gostaria de saber seos colégios 
inscritos na OBM estão automaticamente inscritos na estadual (colegios do Rio de 
Janeiro claro)
Obrigado.

[]'s MP


Re: Triângulo órtico

2000-08-24 Por tôpico Marcos Paulo



Triângulo órtico é o triângulo formado pelos pés 
das alturas de um triângulo acutângulo.
[]'sMP

  - Original Message - 
  From: 
  Pedro 
  
  To: [EMAIL PROTECTED] 
  Sent: Thursday, August 24, 2000 6:03 
  PM
  Subject: Triângulo órtico 
  
   Boa noite, gostaria que alguém disponível pudesse me 
  esclarecer o que é um triângulo órtico, muito obrigado. 
  []' Pedro 
  (3o série do ensino médio)


Re: OBM

2000-08-29 Por tôpico Marcos Paulo



14 horas (horario de Brasilia)

[]'s MP

  - Original Message - 
  From: 
  Jorge Peixoto Morais 
  To: [EMAIL PROTECTED] 
  Sent: Tuesday, August 29, 2000 3:45 
  PM
  Subject: OBM
  
  Qual o horário da 2ª fase da OBM nível 2 neste 
  ano? 


Propriedades tautocrona e Braquistocrona da cicloide

2000-09-06 Por tôpico Marcos Paulo



Alguem conhece uma demonstracao de que a curva 
tautocrona (e braquistocrona) é um aroc de cicloide? Eu vi a pouco tempo uma 
demonstracao que usava transformada de Laplace .. esta, no entanto, nao me 
interessa ... se alguem puder ajudar me mostrando outra demonstracao 
agradeco.

[]'s MP


Re: estranho

2000-09-12 Por tôpico Marcos Paulo

Um conjunto eh enumeravel quando eh possivel estabelecer uma bijecao entre o
conjunto dado e o conjunto dos numeros naturais. Isto acontece com o
conjunto dos racionais mas nao com o conjunto dos reais. Acredito que por
isso pode-se dizer que o infinito dos reais eh  "maior" que o infinito dos
racionais.

Quanto ao seu problema temos:
S = 1 +2/2 +3/4 +4/8 +5/16 + ...   (1)
S/2 = 1/2 + 2/4 + 3/8 + 4/16 + 5/32 +    (2)


Fazendo (1) - (2) teremos:

S/2 = 1 + 1/2 + 1/4 + 1/8 + 1/16 + ...
S/2 = 1/(1/2) = 2 logo S = 4.

[]'s MP
- Original Message -
From: "Eduardo Favarão Botelho" [EMAIL PROTECTED]
To: [EMAIL PROTECTED]
Sent: Monday, September 11, 2000 11:00 PM
Subject: estranho


 Espera aí!

 Que negócio é esso de que um infinito é maior que o outro? como assim
 ser Q um conjunto enumerável?
 Estou confuso.
 E aproveitando a deixa, gostaria de deixar um problema bonitinho:
 calcule S, sendo

 S = 1 +2/2 +3/4 +4/8 +5/16 + ...

 Abraços, Eduardo


 Um exemplo:
 tome o conjunto dos números reais R.
 lembre-se que Q (conjunto dos numeros racionais) e I (conjunto dos
numeros
 irracionais) estao contidos em R.
 Escolha um elemento de R aleatoriamente.
 Sabe qual e a probabilidade desse elemento ser racional?
 ZERO, apesar de Q ser um conjunto infinito e denso em R e portanto esse
 evento e perfeitamente possivel.
 Isto decorre do fato de Q ser um conjunto enumeravel (se e que isso faz
 algum sentido para voce) e I, assim como R nao sao enumeraveis, ou seja
sao
 "muito maiores".





Propriedades tautocrona e Braquistocrona da cicloide

2000-09-12 Por tôpico Marcos Paulo



Desculpem reenviar esta msg mas acredito que qdo 
enviei pela primeira vez havia muitas questoes ssobre a olimpiada Brasileira ... 
reenvio na esperanca de que alguem posa me ajudar. obrigado.

Alguem conhece uma demonstracao de que a curva com 
a propriedade tautocrona (e braquistocrona) é um arco de cicloide? Eu vi a pouco 
tempo uma demonstracao que usava transformada de Laplace .. esta, no entanto, 
nao me interessa ... se alguem puder ajudar me mostrando outra 
demonstracao ou alguma bibliografia agradeco.

[]'s MP


Re: GP

2000-10-10 Por tôpico Marcos Paulo

Eu havia feito igual o Wagner soh que acho que tem outra maneira tb. Olha 
soh: Pegue o angulo BMC (que vale 40) e trace sua bissetriz suponha que F 
seja o ponto de intersecao de BM com Essa bissetriz e D o ponto de 
intersecao da bissetriz com a lado BC. Ai basta provar que o triangulo DFC 
eh semelhante ao triangulo FMN. Assim o Angulo FMN sera de 30 graus e sera 
igual a 20 + o angulo desejado.
Nao parei pra fazer a demonstracao da semelhanca mas jah vi uns tres 
rtiangulos semelhantes ... acho que essa demoinstracao sera possivel. se 
alguem conseguir comunique por favor.

[]'s MP
At 22:10 11/10/00 -0700, you wrote:


--
From: "josimat" [EMAIL PROTECTED]
To: [EMAIL PROTECTED]
Subject: Re: GP
Date: Sun, Oct 8, 2000, 18:24

Olá Wagner!
Tem razão, CBM=60 graus, e não CBN. Desculpem-me pela falha na digitação. 
Eis o enunciado corrigido:

Dado um triângulo ABC, com AB=AC. Tomam-se os pontos N e M pertencentes, 
respectivamente, aos lados AB e AC. Sendo a medida do ângulo BCN=30 graus, 
CBM=60 graus, NBM=20 graus. Determine a medida do ângulo BMN.

[]'s JOSIMAR



Oi Josimar: vejamos uma solução braçal do seu problema.

Seja P a interseção de BM com CN, e façamos BC = 2 para facilitar.  Temos 
então PB = 1 e PC = sqrt(3).
Sejam PM = x e PN = y.

y = tg20 e sqrt(3)/x = tg40. Multiplicando, obtemos y/x = tg20.tg30.tg40.

Acontece que tg20.tg30.tg40 = tg10, o que mostra que o ângulo BMN é igual 
a 10 graus.

Resolvido o problema, resta demonstrar a identidade acima. Isto demanda um 
certo trabalho
algébrico. Faça tg x = t e calcule tg(3x). Voce vai encontrar tg(3x) = t(3 
- t^2)/(1 - t^2).

Em seguida, demonstre a identidade tg(30 - a). tg(3a) . tg(30 + a) = tg a. 
Fazendo a = 10 graus,
fica tudo resolvido.

Fico devendo uma solução mais elegante para seu problema.

Um abraço,
 Wagner.











Re: Solucao nao bracal

2000-10-13 Por tôpico Marcos Paulo

Tente uma generalizacao da Formula de Newton ...

1*S(3) -5*S(2) +4S(1) -5S(0) = 0

Onde S(k) = m^k + n^k + p^k

No NOsso caso: S(0) = 3 (m^0 + n^0 + p^0)
S(1) = 5 (m + n + p)
S(2) = 25 - 2(-4) = 33 (m^2+n^2+p^2 = (m +n +p)^2 - 2(mn + np + mp) )
S(3) = 5 * 33 - 4 *5 + 5 *3 = 165 - 20 + 15 = 160

mnp = 5 neh?? a resposta sera entao 160/5 = 32  (verifiquem os meus 
calculos plz ...)

[]'s MP


Obs formula de newton:

Definimos S(n) = p^n + q^n ( onde p e q sao raizes da equacao Ax^2 +Bx + C =0)

Como p e q sao raizes temos: Ap^2 + Bp +C = 0 (1)  e Aq^2 + Bq + C = 0 (2)

Multiplicando (1) por  p(n -2) e (2) por q^(n-2) e somando os resultados 
vem que:
AS(n) + BS(n-1) + CS(n-2) = 0


para tres fica facil tb 


At 19:05 13/10/00 -0300, you wrote:
Galera,
Entrei recentemente na lista e, lendo sobre solução braçal de problema, 
lembrei-me de uma questão de vestibular:
(UCB)
"A equação x^3 - 5x^2 + 4x - 5 = 0 tem raízes iguais a m, n e p. Determine 
o valor de m^2/np + n^2/mp + p^2/mn."

A solução braçal seria desenvolver a expressão (m+n+p)^3 para isolar m^3 + 
n^3 + p^3. Qual seria a solução não-braçal?

Abraços,
Pedro




Re soma

2000-10-16 Por tôpico Marcos Paulo

Acredito que este problema já tenha sido discutido nesta lista. No entanto,
lá vai:

Usaremos a seguinte propriedade: (k+1)^3 - k^3 = 3k^2 + 3k + 1

1^3 = 1
2^3 - 1^3 = 3*1^2 + 3*1 + 1
3^3 - 2^3 = 3*2^2 + 3*2 + 1
4^3 - 3^3 = 3*3^2 + 3*3 + 1
...
(n+1)^3 - n^3 = 3n^2 + 3n + 1

Somando-se membro a membro temos:

(n+1)^3 = 3*S + 3(1 + 2 + 3 + ... + n) + n

Resolvendo esta equação em S temos: S = n(n+1)(2n+1)/6






Re: pg

2000-11-28 Por tôpico Marcos Paulo



Bom resolver eu resolvi 

a + aq + aq^2 + aq^3 + aq^4 = 211

a(1 + q + q^2 + q^3 + q^4) = 211
Aqui eu pensei que só poderia ter a = 1, mas logo 
vi que não levaria a lugar algum ...

fiz q = r/s
s^4 | a = a = n*s^4 

Como a  100 = a = n*2^4 ou a = 
3^4.

Pela facilidade comecei pelo a = 3^4

q = r/3

daí foi só verificar que funciona para r 
=2.

a sequencia fica:

81, 54, 36, 6, 4 

A resposta 121.

Deve haver uma maneira mais formal de fazer ... 
espero vê-la aki.

[]'s MP



  - Original Message - 
  From: 
  josimat 
  To: [EMAIL PROTECTED] 
  Sent: Monday, November 27, 2000 11:52 
  PM
  Subject: pg
  
  Olá pessoal!
  Acabei de receber por 
  telefone:
  A soma dos 5 termos, todos inteiros e menores 
  que 100, de uma PG é 211. Calcule a soma dos termos quadrados perfeitos dessa 
  PG (que só possui esses 5 termos).
  []'s 
JOSIMAR


Re: Análise combinátoria.

2000-11-28 Por tôpico Marcos Paulo


- Original Message -
From: mcddj [EMAIL PROTECTED]
To: [EMAIL PROTECTED]
Sent: Tuesday, November 28, 2000 9:25 PM
Subject: Análise combinátoria.


 Preciso de ajuda, urgente. Quem puder me socorrer
 agradeço.


 1. Em um corredor há 900 armários, numerados de 1 a 900,
 inicialmente todos fechados. 900 pessoas, numeradas de 1
 a 900, atravessam o corredor. A pessoa de número k
 reverte o estado de todos os armários cujos números sâo
 múltiplos de k. Por exemplo, a pessoa de número 4 mexe
 nos armários de números 4, 8, 12,..., abrindo os que
 encontra fechados e fechando os que encontra abertos. ao
 final, quais armários ficarão abertos?

Basta verificar que a porta ficará aberto somente se for "mexido" uma
quantidade ímpar de vezes.
O Armario será mexido tantas vezes quantos forem seus divisores. então
ficarão abertos os armarios
com quantidade ímpar de divisores.

Número de divisores de um número dado:

N = a ^x * b^y * c^z ...   com a, b, c... primos e x, y, z... pertencente
aos naturais
d(N) = (x+1)(y+1)(z+1)...

Queremos então que o produtos dos consecutivos dos expoentes dos números
seja ímpar. Isso soh
ocorrerá se todos esses concecutivos forem ímpares e portanto os expoentes
devem ser PARES.

portanto os armarios que ficarão abertos são os de número igual a um
quadrado perfeito.



 2. Um vagão de metrô tem 10 bancos individuais, sendo 5
 de frente e 5 de costas. De 10 passageiros, 4 preferem
 sentar de frente, 3 preferem sentar de costas e os
 demais não têm preferência. De quantos modos eles podem
 se sentar, respeitadas as preferências?

C5,4 * C5, 3 * 3!

Acho q eh isso ...



 3. De um baralho comum de 52 cartas, sacam-se
 sucessivamente e sem reposição duas cartas. De quantos
 modos isso pode ser feito se a primeira carta deve ser
 de copas e a segunda não deve ser um rei?
Dividindo em 2 casos:

1 Rei de copas e qq carta diferente de rei = 1 * 48
carta de copas (sem rei) e qq carta diferente de rei = 12 * 48

A resposta seria a soma ...


 4. Escrevem-se números de 5 dígitos, inclusive os
 começados em 0, em cartões. Como 0, 1, e 8 não se
 alteram de cabeça para baixo e como6, de cabeça para
 baixo, se transforma em 9 e vice-versa, um mesmo cartão
 pode representar dois números (por exemplo, 06198 e
 86190). Qual é o número mínimo de cartões para
 representar todos os números de 5 dígitos?





 Abraços...




[]'s MP

 __
 Preocupado com vírus? Crie seu e-mail grátis do BOL com antivírus !
 http://www.bol.com.br







Re: Análise combinátoria.

2000-11-29 Por tôpico Marcos Paulo

Obrigado pelas correções
: - )

[]'s MP
- Original Message -
From: Carlos Stein Naves de Brito [EMAIL PROTECTED]
To: [EMAIL PROTECTED]
Sent: Wednesday, November 29, 2000 10:32 PM
Subject: Re: Análise combinátoria.




  From: "Marcos Paulo" [EMAIL PROTECTED]
  Reply-To: [EMAIL PROTECTED]
  Date: Tue, 28 Nov 2000 23:45:14 -0200
  To: [EMAIL PROTECTED]
  Subject: Re: Análise combinátoria.
 
 
  - Original Message -
  From: mcddj [EMAIL PROTECTED]
  To: [EMAIL PROTECTED]
  Sent: Tuesday, November 28, 2000 9:25 PM
  Subject: Análise combinátoria.
 
 
  Preciso de ajuda, urgente. Quem puder me socorrer
  agradeço.
 
 
  1. Em um corredor há 900 armários, numerados de 1 a 900,
  inicialmente todos fechados. 900 pessoas, numeradas de 1
  a 900, atravessam o corredor. A pessoa de número k
  reverte o estado de todos os armários cujos números sâo
  múltiplos de k. Por exemplo, a pessoa de número 4 mexe
  nos armários de números 4, 8, 12,..., abrindo os que
  encontra fechados e fechando os que encontra abertos. ao
  final, quais armários ficarão abertos?
 
  Basta verificar que a porta ficará aberto somente se for "mexido" uma
  quantidade ímpar de vezes.
  O Armario será mexido tantas vezes quantos forem seus divisores. então
  ficarão abertos os armarios
  com quantidade ímpar de divisores.
 
  Número de divisores de um número dado:
 
  N = a ^x * b^y * c^z ...   com a, b, c... primos e x, y, z...
pertencente
  aos naturais
  d(N) = (x+1)(y+1)(z+1)...
 
  Queremos então que o produtos dos consecutivos dos expoentes dos números
  seja ímpar. Isso soh
  ocorrerá se todos esses concecutivos forem ímpares e portanto os
expoentes
  devem ser PARES.
 
  portanto os armarios que ficarão abertos são os de número igual a um
  quadrado perfeito.
 
 
 
  2. Um vagão de metrô tem 10 bancos individuais, sendo 5
  de frente e 5 de costas. De 10 passageiros, 4 preferem
  sentar de frente, 3 preferem sentar de costas e os
  demais não têm preferência. De quantos modos eles podem
  se sentar, respeitadas as preferências?
 
  C5,4 * C5, 3 * 3!
 
  Acho q eh isso ...
 nao é combinacao, pois se mudar de posicao, muda o exemplo, é so trocar C
 por arranjo, se nao me engano.
 
 
 
  3. De um baralho comum de 52 cartas, sacam-se
  sucessivamente e sem reposição duas cartas. De quantos
  modos isso pode ser feito se a primeira carta deve ser
  de copas e a segunda não deve ser um rei?
  Dividindo em 2 casos:
 
  1 Rei de copas e qq carta diferente de rei = 1 * 48
  carta de copas (sem rei) e qq carta diferente de rei = 12 * 48
 
 se tirou uma carta diferente de rei, sobram 47 cartas diferentes de rei,
 logo em vez de 12*48 seria 12*47
  A resposta seria a soma ...
 
 
  4. Escrevem-se números de 5 dígitos, inclusive os
  começados em 0, em cartões. Como 0, 1, e 8 não se
  alteram de cabeça para baixo e como6, de cabeça para
  baixo, se transforma em 9 e vice-versa, um mesmo cartão
  pode representar dois números (por exemplo, 06198 e
  86190). Qual é o número mínimo de cartões para
  representar todos os números de 5 dígitos?
 
 
 
 
 
  Abraços...
 
 
 
 
  []'s MP
 
 
__
  Preocupado com vírus? Crie seu e-mail grátis do BOL com antivírus !
  http://www.bol.com.br
 
 
 
 





Re: Divisíveis

2000-12-10 Por tôpico Marcos Paulo


- Original Message -
From: "João Paulo Paterniani da Silva" [EMAIL PROTECTED]
To: [EMAIL PROTECTED]
Sent: Sunday, December 10, 2000 2:28 PM
Subject: Divisíveis



 Olá. Por que um número é divisível por 3 se a soma de seus algarismos
é
 divisível por 3?

um número qq decomposto em unidades fica assim:

N =A_0*10^n  + A_1*10^(n-1) + ... + A_n

Mas 10 = 3 q + 1 = 10 ^k = 3R + 1

Daí N = A_0(3q_0 + 1) + A_1(3q_1+1) + ... + A_n.

N = 3 (/\/\/\/\/\\/\/\/\/\/\) + A_0 + A_1 + A_2 + ...+ A_n . Como a primeira
parcela jah eh multipla de 3 basta q a soma das outras seja multpla de 3 tb.



 E um número é divisível por 7 se o número formado por todos seus
 algarismos, exceto o último, menos o dobro do último é divisível por 7?
 Como se prova isto?
 Fui claro?

Fazer esse com um exemplo:

N = ABCD o q vc mandou fazer foi ABC - 2*D neh? pensa no q vc ta fazendo:

Eh o mesmo q ABCD - (20*D + D) se vc vai subtraindo um multiplos de 21 do
numero e em algum momento vc encontra um multiplo de 7 eh pq o numero todo
eh multiplo de 7 pois 21 tb eh multiplo de 7.
Esse no entanto não eh o melhor criterio de divisibilidade por 7 pois não
nos fornece o resto da divisao por 7 do numero.
c deve conhecer o outro das classes (soma das classes impares - soma das
classes pares e tal) esse da o resto.

Espero ter ajudado.

[]'MP








 João Paulo Paterniani da Silva



_
 Get more from the Web.  FREE MSN Explorer download :
http://explorer.msn.com





Re: Trigonometria-Ajuda Urgente

2000-12-13 Por tôpico Marcos Paulo

Multiplique e divida pra não alterar a expressão.
 : - )
[]'s MP
- Original Message -
From: "Augusto Morgado" [EMAIL PROTECTED]
To: [EMAIL PROTECTED]
Sent: Wednesday, December 13, 2000 6:32 AM
Subject: Re: Trigonometria-Ajuda Urgente


 Multiplique por sen(pi/65). Use varias vezes que sena.cosa=(sen2a)/2.
 Lembre-se ao final que pi/65 e 64pi/65 sao suplementares e tem o mesmo
 seno.

  Hugo Iver Vasconcelos Goncalves wrote:
 
  Olá, preciso de ajuda nesta questão, se possível ainda para hoje...
  desde já agradeço vossa ajuda,
 
  Prove que:
 
cos(Pi/65)*cos(2Pi/65)*cos(4Pi/65)*cos(8Pi/65)*cos(16Pi/65)*cos(32Pi/65)=1/6
4




Re: Quadratura do círculo

2000-12-19 Por tôpico Marcos Paulo



Quer dizer construir um quadrado com a mesma area 
de um círculo usando apenas regua e compasso.
Este termo ficou conhecido como sinonimo de 
impossível.
[]' s MP

  - Original Message - 
  From: 
  Davidson 
  Estanislau 
  To: obm 
  Sent: Tuesday, December 19, 2000 4:02 
  PM
  Subject: Quadratura do círculo
  
  O que quer dizer: A quadratura de um 
  círculo?
  
  Davidson


Re: problema do triângulo.

2001-02-13 Por tôpico Marcos Paulo

Trace BCR (com r pertencendo ao lado AB) e vc encontrara um monte de
triangulos isosceles (inclusive um equilatero)... ai fica fcil...
[]'s MP
- Original Message -
From: "Exercicio" [EMAIL PROTECTED]
To: [EMAIL PROTECTED]
Sent: Monday, February 12, 2001 11:47 PM
Subject: Re: problema do tringulo.





  Certinho amigo!

  O erro  todo meu... Desenhei de modo totalmente errado a figura

  Gostaria de aproveitar essa msg e declarar:

  ESQUEAM A FIGURA ANTERIOR.!

  Desculpe-me pelo erro...

  A figura certa vai nesse email

 Falow's

   Exercicio 

  http://exercicio.cjb.net
  ICQ # 102856897



 Bruno Furlan escreveu:

  No pode ser isso, tem erro a...
  Se for isso, com AB=AC como est escrito embaixo da figura,
ABC=ACB=80,
  da BCB'=50, de onde tiramos BOC=80 e conseqentemente C'OB=100, e
  B'BO=30. Assim, o ngulo destacado em verde mede 50.
 
  ("legenda": B'  o ponto onde se encontram AC e a ceviana que sai de B;
C' 
  o ponto onde se encontram AB e a ceviana que sai de C; O  o encontro
das
  duas cevianas.)
 

 --
--
  















Re: Intuicao na Geometria Analitica

2001-03-21 Por tôpico Marcos Paulo



Se s forma um angulo alfa com o eixo x e s forma um 
angulo beta com o memso eixo x, tem se q o angulo entre as duas retas eh 
dado por # = (alfa - beta) aplique tangente nos dois lados da igualdade e 
vc tera a formula q vc escreveu 
[]'s MP

  - Original Message - 
  From: 
  Gustavo 
  Martins 
  To: [EMAIL PROTECTED] 
  Sent: Wednesday, March 21, 2001 7:47 
  PM
  Subject: Intuicao na Geometria 
  Analitica
  
  Considere os seguintes dados:
   mr = coeficiente angular da reta r;
   ms = coeficiente angular da reta s;
   mt = coeficiente angular do angulo agudo formado pelo 
  encontro das retas r e s;
  # = angulo agudo formado pelo encontro das retas 
  r e s;
   x indica multiplicação;
   tg# = |ms - mr/(1 + msxmr)|.
  
  Como faco para concluir essa equacao intuitivamente? Ja conseguiuma 
  explicacao intuitiva sobreoutras equações de Geometria Analitica, mas 
  nao consigo fazer o mesmo com essa.
  
  Obrigado,
  Gustavo
  
  


duvidas crueis ...

2001-03-21 Por tôpico Marcos Paulo



Hj me levaram um problema pra sala de aula e 
disseram q está na revista Eureka.. o problema eh assim:

Seja N um número tal que d(2N^2) = 28 e d(3N^2) =30 
determine d(6N^2) onde d(X) = número de divisres de X. Eu resolvi 
assim:
Seja N = 2^a * 3^b * 5^c * ... * p^k 
(onde p é um primo qualquer e a, b, c, ..., k são inteiros não 
negativos)
Usando os dados escrevi que

d(2N^2) = (2a +2)(2b+1)(2c+1)(2d+1)...(2k+1) = 
28
d(6N^2) = (2a +2)(2b+2)(2c+1)(2d+1)...(2k+1) = (2a 
+2)(2b+2) * 28/[(2a+2)(2b+1)]=
= (2b+2)/(2b+1)*28 =[ (2b+1)/(2b+1) + 
1/(2b+1)] * 28 = 28 + 28/(2b+1).

Como o numero de divisores eh inteiro, temos q a 
parcela 28/(2b+1) deve ser inteira e portanto conclui que 
2b + 1 = 1 ou 2b + 1 = 7, ou seja d(6N^2) = 
58 (para b = 0) ou d(6N^2) = 32 (parab = 3)

repetindo esse raciocinio para d(3N^2) = 30, 
obtem-se interseção unicapara d(6N^2) = 32 e portanto conclui q esse eh o 
numero procurado.

A solução está correta? há solução mais formal? eu 
esqueci alguma propriedade importante?

Agradeço desde já a atenção.

[]'s MP


Re: Ajuda

2001-03-22 Por tôpico Marcos Paulo



Dizer q A = B mod d é dizer q A e B deixam o mesmo 
resto na divisão por d, ou ainda q A - B é multiplo de d
[]'s MP

  - Original Message - 
  From: 
  Gustavo 
  Martins 
  To: [EMAIL PROTECTED] 
  Sent: Wednesday, March 21, 2001 7:10 
  PM
  Subject: Re: Ajuda
  
  Olhei a resolução do problema do Igor, mas não sei o que é mod. Alguem 
  pode me explicar?
  
  Atenciosamente,
  Gustavo
  
- Original Message - 
From: 
Rodrigo 
Villard Milet 
To: [EMAIL PROTECTED] 
Sent: Wednesday, March 21, 2001 1:21 
PM
Subject: Re: Ajuda

1) 2^n + 1 = 0 mod3 implica 2^n = -1mod3, logo (-1)^n = -1mod3, então n 
é ímpar.

2) x^2 + 3x + 2 = (x+1)*(x+2). Note q esse número é sempre par, pois é 
produto de dois consecutivos. Logo, basta achar os x, para os quais E = 
(x+1)*(x+2) é múltiplo de 3. Para isso, calcule quantos são os x, para os 
quais 3 não divide E, os seja, 3 divide x. De 0 a 25, há 9. Logo, há 26 - 9 
= 17 x`s, para os quais 3 divide E, e por conseguinte, 6 
divide E.

 ¡ Villard 
!

  -Mensagem original-De: 
  Igor Castro [EMAIL PROTECTED]Para: 
  [EMAIL PROTECTED] [EMAIL PROTECTED]Data: 
  Terça-feira, 20 de Março de 2001 22:14Assunto: 
  Ajuda
  Caríssimos colegas, estou precisando de ajuda 
  nos seguintes problemas: (parecem simples)
  
  
  1) Determine n natural para que 2^n + 1 seja 
  divisível por 3. (resolver algebricamente)
  
  2) Se x pertence à {0,1,2,...,25), para 
  quantos valores de x, x^2 +3x +2 é múltiplo de 6?
  
  Estava resolvendo esses problemas num 
  capítulo de divisibilidade e congruências, se puderem usar só o 
  
  conceito básicos dessas teorias, 
  agradeço.
  
  Igor Castro
  


Re: regra de tres

2001-03-25 Por tôpico Marcos Paulo

Se a plantao maior foi cortada por todo pessoal em meio dia e por metade
do pessoal na outra metade do dia, ento temos que a metade do pessoal
cortou 1/3 da plantao. Como o campo menor foi cortado pelo mesmo pessoal e
tem metade do  tamanho do maior ento ficou por ser cortado no dia seguinte:
1/2 - 1/3 da plantao maior. agora ficou simples ...  1 trabalhador corta
1/6 da plantao maior por dia ... o total  cortado no primeiro dia foi  6/6
+ 2/6 = 8/6 portanto  8 trabalhadores fizeram o trabalho.

algo errado nisso?

[]'s MP
- Original Message -
From: "Rodrigo Villard Milet" [EMAIL PROTECTED]
To: [EMAIL PROTECTED]
Sent: Sunday, March 25, 2001 3:15 AM
Subject: Re: regra de tres


 Seja x o nmero de cortadores de cana e A a rea da menor plantao e
 considere t o tempo de trabalho num dia inteiro. Logo, temos :
 (t/2)*x + (t/2)*(x/2) = 2A (I), pois durante t/2 hrs, todos trabalharam e
 nas outras t/2 hrs a outra metade trabalhou. De (I), temos 3xt/4 = 2A e
 assim, xt/4 = 2A/3.
 Note agora, que (t/2)*(x/2) = A - d (II), onde d  o pedao q falta. Mas d
=
 1*t=t, pois um trabalhador cortou a cana num dia todo. Logo, de (II) :
 tx/4 = A - t. Como tx/4 = 2A/3, temos 2A/3 = A - t que implica t = A/3
 Como xt/4 = 2A/3 = 2t, x/4 = 2, ou seja, h 8 trabalhadores !!!

  Abraos,
   Villard !
 -Mensagem original-
 De: Erico Furukawa [EMAIL PROTECTED]
 Para: [EMAIL PROTECTED] [EMAIL PROTECTED]
 Data: Domingo, 25 de Maro de 2001 02:39
 Assunto: regra de tres


 Meu nome e Erico e eu queria um favor de voces.
 Queria saber a resolucao de um problema:
 
 "Uma turma de cortadores de cana deveria trabalhar em
 duas plantacoes, uma com o dobro da area da outra.
 Durante meio dia, todos trabalharam na plantacao
 maior. Depois do almoco, metade da turma continuou na
 plantacao grande, enquanto a outra metade passou para
 a menor. No fim da tarde, o trabalhado estava quase
 terminado, faltando apenas uma pequena faixa da
 plantacao menor. Esse pedaco foi concluido por um
 unico trabalhador, que cortou cana durante todo o dia
 seguinte. Quantos cortadores de cana havia na turma?"
 
 
 
 O YAHOO! GEOCITIES CHEGOU AO BRASIL!
 Crie sua home page com tudo em portugus - http://br.geocities.com
 





Re: QUESTÃO DESAFIO 1 BY João cláudio

2001-05-14 Por tôpico Marcos Paulo

A algum tempo eu postei uma identidade conhecida (?) q resolve o problema
...

tg(60 - x) * tg x * tg (60 + x) = tg (3x)

não eh tão dificil de demonstrar ...

[]'s MP
- Original Message -
From: joao claudio [EMAIL PROTECTED]
To: OBM [EMAIL PROTECTED]
Sent: Saturday, May 12, 2001 6:05 PM
Subject: QUESTÃO DESAFIO 1 BY João cláudio


 resolvam a seguinte questão:

 a)tg20.tg40.tg80





 ---
 Visite: http://www.email.com.br
 O melhor servigo de webmail gratuito.




[obm-l] Re: [obm-l] Questão envolvendo tráfego de veículos

2003-01-13 Por tôpico Marcos Paulo



Pense na quantidade de carros que sairão por X. 
Note que a saida x perde carros sempre que há uma esquina com entrada para 
direita. Há apenas 2 esquinas deste tipo e portanto na primeira a saida x perde 
256 carros (sobrando 256) e na segunda ela perde masi 128 ficando com apenas 128 
carros. O restante dos carros sairá por Y. Resposta: 512 - 128 = 
384.
A solução que vc apresentou é exatamente igual a 
esta sendo que o 1 representa o total de carros (q eu chamei de 512) e o 1/4 a 
quantidade de carros que sairão por X (q eu chamei de 128).O total de 
carros menos o que sai por x engloba os q tomaram o caminho "inferior" . 

[]'s MP

  - Original Message - 
  From: 
  [EMAIL PROTECTED] 
  
  To: [EMAIL PROTECTED] 
  Sent: Monday, January 13, 2003 8:07 
  PM
  Subject: [obm-l] Questão envolvendo 
  tráfego de veículos
  Olá pessoal, 
  (CESGRANRIO) A figura abaixo (clique aqui para 
  ver) representa uma área de ruas de mão única. Em cada esquina em que há 
  duas opções de direção (vide figura) o 
  tráfego se divide igualmente entre elas. Se 512 carros entram na área por P, 
  determine o número dos que vão sair por Y. Solução: O tráfego em 
  P se divide em dois. Então, ½ da quantidade dos carros seguem em 
  direção a X.. Em seguida, o tráfego se divide em dois novamente. 
  Portanto, na saída X teremos a metade da metade dos carros, ou seja , 
  teremos ½ × ½ = ¼. Assim, na saída Y teremos o total de carros menos 
  ¼ do total, ou seja, teremos 1 - ¼ = 3/4 dos carros. Se entram 512 
  carros em P, então em Y vão sair 3/4 de 512 = 3/4 × 512 = 384 
  carros. Obs: A figura e a questão pode ser vista no site: 
  http://www.ezequiassilva.hpg.ig.com.br/inteiros.html 
  Dúvida: O que eu não entendi, foi por quê na resolução 
  somente foi considerado somente o caminho, ou o destino da metade que partindo 
  de P chega até Y pelo caminho superior , por quê não foi considerado o caminho 
  inferior, ou seja, a outra metade de 512 (ou seja, 256) que chegou a Y pelo 
  caminho inferior passando parte destes carros nas ruas secundárias ? 



Re: [obm-l] probabilidade

2003-01-14 Por tôpico Marcos Paulo



Neste caso a quantidade de resultados possíveis é 
bem pequena e portanto é possível lista-los.
Seja A a vitória da primeira equipe e B a vitória 
da segunda equipe. Os resultados possíveis são:
ABABA
ABAA
AA
BAA
BABAA
BABAB
BABB
BB
ABB
ABABB

dentre as 10 possibilidades listadas há 8 em que 
uma das equipes tem 2 vitórias consecutivas (80%).

Tenho ainda uma dúvida quanto a este "uma 
equipe".Se "uma equipe" for o mesmo que "alguma equipe" então a resposta é 
a dada acima. 
Se, por outro lado, "uma equipe" for o mesmo que 
"uma determinada equipe" então a resposta será metade da resposta 
dada.
Para deixar mais claro o que não entendi vou 
modificar o enunciado e gostaria que me respondessem se o enunciado é 
equivalente ao pedido nesta mensagem.
"Palmeiras e Botafogo disputam um torneio de 
futebol em que saírá vencedor aquele que obtiver 2 vitórias consecutivas ou 3 
vitórias alternadas. Se não é possível o empate (disputa de penalti), qual a 
probabilidade de que o Botafogo saia vitorioso do torneio com duas vitórias 
consecutivas?"
Obs. A escolha dos times não foi aleatória .. 
procurei times que tivessem igualdade de condições na disputa de qualquer 
partida, ou seja, a vitória de qualquer um dos times contece com 50% de 
probabilidade. Nunca iria colocar, por exemplo, Flamengo e Botafogo visto 
que estes estão em categorias diferentes e portanto a vitória do Flamengo 
aconteceria com maior probabilidade.
Não sei se minha dúvida procede mas resolvi 
responder apenas para tirar esta dúvida.

[]'s MP

  - Original Message - 
  From: 
  [EMAIL PROTECTED] 
  
  To: [EMAIL PROTECTED] 
  Sent: Monday, January 13, 2003 11:36 
  PM
  Subject: [obm-l] probabilidade
  Duas equipes disputam entre si uma série de jogos em que 
  não pode ocorrer empate e as duas equipes têm as mesmas chances de vitória. A 
  primeira equipe que conseguir duas vitórias seguidas ou três vitórias 
  alternadas vence a série de jogos. Qual a probabilidade de uma equipe vencer a 
  série de jogos com duas vitórias 
seguidas?


[obm-l] Re: [obm-l] análise combinatória

2003-01-14 Por tôpico Marcos Paulo



Um códogo é determinado pela escolha das cores de 6 
barras a primeira barra pode ser escolhida de 2 cores, a segunda pode ser 
escolhida de 2 cores e assim por diante até a 6a barra que pode ser escolhida de 
2 cores. Como a escolha da cor de uma barra não interfere na escolha da cor das 
outras barras o total de códigos será o produto 2^6 = 64. Porém o enunciado 
descarta a possibilidade de um código conter todas asbarras brancas e 
todas as barras pretas portanto do total devemos descontar estas duas 
possibilidades e a resposta fica então 64 - 2 = 62.
A "fórmula" que vc colocou na mensagem original dá 
o total de maneiras que vc pode escolher p objetos dentre n e nào tem nada a ver 
com o exercício.

[]'s MP

  - Original Message - 
  From: 
  [EMAIL PROTECTED] 
  
  To: [EMAIL PROTECTED] 
  Sent: Tuesday, January 14, 2003 9:41 
  PM
  Subject: [obm-l] análise 
  combinatória
  Olá pessoal, 
  Alguém consegue resolver estre problema de análise combinatória: 
  (U.C SALVADOR) Um código para leitura ótica é constituído por 6 barras 
  brancas ou pretas. Nenhum código tem barras de uma só cor. Veja dois exemplos 
  desses códigos: Obs: Vou descrever como são estes exemplos: 
  Imagine dois retângulos, em que cada um é formado por 6 listas 
  verticais, para facilitar a descrição vamos ordenar as listas, ou seja, a 1º 
  (da esquerda para direita), depois 2º...6º lista. Imagine que o primeiro 
  retangulo esta pintado assim: 2º lista e 5º lista (ambas de preto) e o 
  restante de branco. Agora, imagine o segundo retangulo (código de barras) com 
  a 1º, 2º e 5º lista sendo pretas e as restantes brancas. Dúvida: Por 
  quê podem ser formados 62 (segundo meu gabarito) códigos, distintos entre si? 
  Eu tentei aplicar cn,p=n!/(n-p)!p! mas não cheguei no resultado. Será que é 
  arranjo?  



Re: [obm-l] geometria espacial

2003-01-14 Por tôpico Marcos Paulo



Quando seccionamos um cone por um plano podemos 
observar dois cones semelhantes (o cone original e o cone que é retirado para 
gerar o tronco). A razão de semelhança é igual a razão entre as alturas e a 
razão entre os volumes é o cubo da razão de semelhança. No seu exercício o cone 
menor tem 1/8 do volume (1 - 7/8) do cone maior e portanto a razão entre os 
cubos das alturas deve ser 1/8, ou seja, (h/H)^3 = 1/8 = h/H = 1/2 como H = 
12 h = 6.
[]'s MP

  - Original Message - 
  From: 
  [EMAIL PROTECTED] 
  
  To: [EMAIL PROTECTED] 
  Sent: Tuesday, January 14, 2003 9:42 
  PM
  Subject: [obm-l] geometria espacial
  Olá pessoal, 
  Alguém consegue me auxiliar nesta questão de geometria espacial? 
  Seccionando-se um cone reto por u plano paralelo à sua base 
  obtém-se um tronco de cone cujo volume é igual a 7/8 do volume do cone 
  original. Se a altura do cone original é de 12 cm, a que distância do vértice 
  está a secção? Resp: 6cm 


[obm-l] geometria espacial (errata)

2003-01-14 Por tôpico Marcos Paulo





  - Original Message - 
  From: 
  Marcos 
  Paulo 
  To: [EMAIL PROTECTED] 
  Sent: Tuesday, January 14, 2003 11:39 
  PM
  Subject: Re: [obm-l] geometria 
  espacial
  
  Quando seccionamos um cone por um plano PARALELO 
  À BASE, podemos observar dois cones semelhantes (o cone original e o cone que 
  é retirado para gerar o tronco). A razão de semelhança é igual a razão entre 
  as alturas e a razão entre os volumes é o cubo da razão de semelhança. No seu 
  exercício o cone menor tem 1/8 do volume (1 - 7/8) do cone maior e portanto a 
  razão entre os cubos das alturas deve ser 1/8, ou seja, (h/H)^3 = 1/8 = 
  h/H = 1/2 como H = 12 h = 6.
  []'s MP
  
- Original Message - 
From: 
[EMAIL PROTECTED] 
To: [EMAIL PROTECTED] 
Sent: Tuesday, January 14, 2003 9:42 
PM
Subject: [obm-l] geometria 
espacial
Olá pessoal, 
Alguém consegue me auxiliar nesta questão de geometria espacial? 
Seccionando-se um cone reto por u plano paralelo à sua base 
obtém-se um tronco de cone cujo volume é igual a 7/8 do volume do cone 
original. Se a altura do cone original é de 12 cm, a que distância do 
vértice está a secção? Resp: 6cm 



[obm-l] Re: [obm-l] sen,cos tg no ciclo trigonométrico

2003-01-21 Por tôpico Marcos Paulo



olá,

Pense na redução deste ângulo ao primeiro 
quadrante. Estando no primeiro quadrante vc pode dizer que este ângulo é um dos 
ângulos de um triângulo pitagórico de lados 3, 4 e 5 unidades de comprimento. o 
seno do ângulo é 4/5 e o cosseno do ângulo é 3/5. Mas como o ângulo é do segundo 
quadrante, o seno será positivo e o cosseno negativo. A resposta será: 3/5 - 4/5 
= -1/5

[]'s MP
- Original Message - 

  From: 
  [EMAIL PROTECTED] 
  
  To: [EMAIL PROTECTED] 
  Sent: Wednesday, January 22, 2003 2:32 
  AM
  Subject: [obm-l] sen,cos tg no ciclo 
  trigonométrico
  Olá pessol, 
  Como resolver está questão que caiu na fuvest: (FUVEST) Se 
  tgx=3/4 e Pi x3pi/2, o valor de cosx-senx é: Resposta:-1/5 
  Eu tentei resolver este exercício transformando a tg em senx/cosx e 
  depois eu tenter aplicar a relação fundamental sen^2x + cos^2x=1 para eliminar 
  estas incóginas e ficar com o 1 e apenas uma mas eu rodeava no exercício e 
  chegava a sen^2x - cos^2x, ou seja com sinal trocado. 



[obm-l] Re: [obm-l] transformação de arcos

2003-01-21 Por tôpico Marcos Paulo



Resolva novamente sua equacão. Afinal, -1 não é 
raiz de x^2 + 2x -1 =0 ((-1)^2 + 2*(-1) - 1 = - 2).
[]'s MP

  - Original Message - 
  From: 
  [EMAIL PROTECTED] 
  
  To: [EMAIL PROTECTED] 
  Sent: Wednesday, January 22, 2003 3:39 
  AM
  Subject: [obm-l] transformação de 
  arcos
  Olá pessoal, 
  (PUC-SP) A tg 22,5º é igual a: Resposta: (sqrt2) - 1 
  Obs: Como 22,5º= 45º-22,5º, eu tentei resolver da seguinte maneira: 
  tg22,5º=tg(45º-22,5)=(tg 45º - tg 22,5º)/(1 + tg 45º*tg 22,5º) 
  tg22,5º= (1 - tg 22,5º)/(1+tg22,5º) , passando o denominador para o 
  primeiro membro temos tg^2 (22,5º) + 2*tg22,5º -1=0. Eu arbitrei tg22,5º=y e 
  facilitamos a eq. do 2º- y^2 + 2y - 1=0 resolvendo eu cheguei ao valor de 
  -1 e como eu arbitrei y=tg22,5º temos tg22,5º= -1 (IMPOSSÍVEL) Onde está meu 
  erro?  


Re: [obm-l] P.A

2003-02-12 Por tôpico Marcos Paulo



olá,
A_1 = b^2 - a^2 = (b-a)(b+a) = r(b+a), visto que 
b-a = r
A_2 = c^2 - b^2 = r(c+b)
A_3 = d^2 - c^2 = r(c+d).
A sequencia A_1, A_2, A_3 será uma PA se as 
diferenças A_2 - A_1 e A_3 - A_2 forem iguais e nesse caso essa diferença será a 
razão.
Fazendo A_2 - A_1, temom:
r(c+b) - r(b + a) = r(c - a). Como a,b,c,d é uma 
PA, c - a = 2r e portanto A_2 - A_1 = r*2r = 2r^2

Fazendo A_3 - A_2, temos
r(d+c) - r(c + b) = r(d - b). Como a,b,c,d é uma 
PA,d -b = 2r e portanto A_3 - A_2 = r*2r = 2r^2, o7u seja, a 
sequencia dada é uma PA de razão 2r^2.

[]'s MP

  - Original Message - 
  From: 
  [EMAIL PROTECTED] 
  
  To: [EMAIL PROTECTED] 
  Sent: Wednesday, February 12, 2003 6:17 
  PM
  Subject: [obm-l] P.A
  Olá pessoal, 
  (UNESP) Se a, b, c e d formam, nesta ordem, uma P.A de razão r, então 
  b^2 - a^2, c^2 - b^2 e d^2 - c^2 formam, nesta ordem, uma P.A de razão: 
  resp: 2*r^2 Dúvida: Percebi que podemos fazer (b-a)* (b+a), 
  (c-b)*(c+b), (d - c)*(d+c). Como é uma P.A [ (c-b)*(c+b)] - [(b-a)* 
  (b+a)] = [(d - c)*(d+c)] - [ (c-b)*(c+b)] = r . A minha dúvida quando 
  tentava resolver esta questão estava nestas somas em parênteses. 



Re: [obm-l] Outra de P.A

2003-02-12 Por tôpico Marcos Paulo



a[6] = a[1] + 15 = b[1]
a[3] = a[1] + 6 = b[1] * 0,5

ou seja, a[6] = 2a[3] o que nos dá a[1] = e 
portanto b[1] = 18


[]'s MP

  - Original Message - 
  From: 
  [EMAIL PROTECTED] 
  
  To: [EMAIL PROTECTED] 
  Sent: Wednesday, February 12, 2003 6:18 
  PM
  Subject: [obm-l] Outra de P.A
  Olá pessoal, 
  Como resolver esta questão: (UF-RS) Sabendo que a[n] é 
  uma P.A de razão 3, b[n] é uma P.G de razão 1/2, a[6] = b[1] e a[3] = b[2], 
  então a[1] + b[1] é : resp: 21 



Re: [obm-l] Provar primo impar

2003-03-22 Por tôpico Marcos Paulo
Todo número (par ou ímpar) pode ser escrito como
4n, 4n + 1, 4n + 2, 4n + 3 (obviamente isso não é privilégio do 4)

Os número pares serão escritos  como 4n ou 4n + 2 e
os ímpares como 4n + 1 e 4n + 3
4n + 3 = 4n + 4 - 1 = 4(n+1) - 1 = 4N - 1

[]'s MP
- Original Message -
From: Carlos Maçaranduba [EMAIL PROTECTED]
To: [EMAIL PROTECTED]
Sent: Saturday, March 22, 2003 9:44 AM
Subject: [obm-l] Provar primo impar


 ei pessoal, como é que eu provo que qualquer número
 primo impar pode ser escrito ou da forma 4n + 1 ou
 4n - 1 ??



 
 =

 ___
 Yahoo! Mail
 O melhor e-mail gratuito da internet: 6MB de espaço, antivírus, acesso
POP3, filtro contra spam.
 http://br.mail.yahoo.com/
 =
 Instruções para entrar na lista, sair da lista e usar a lista em
 http://www.mat.puc-rio.br/~nicolau/olimp/obm-l.html
 O administrador desta lista é [EMAIL PROTECTED]
 =

=
Instruções para entrar na lista, sair da lista e usar a lista em
http://www.mat.puc-rio.br/~nicolau/olimp/obm-l.html
O administrador desta lista é [EMAIL PROTECTED]
=


RES: [obm-l] ajuda geometria

2003-06-01 Por tôpico Marcos Paulo
Title: Mensagem



Trace 
os segmentos MC e BP. Observe que o triâgulo NMC é retângulo já que M e B são 
diametralmente opostos (o ângulo MCB está inscrito num arco de meia volta). Como 
o triângulo BPN é semelhante ao NMC (caso A-A) NPB é reto. O arco q 
subtendea corda MC é um arco de 60º e portanto med(MC) = R. Temos 
ainda que med(NC) = R*SQRT(3)/2 e usando o teorema de pitagoras vc descobre que 
med (NM) = R*SQRT(7)/2. Agora basta usar a semelhança e descobrir med(NP),que, 
se eu não errei todas as contas (o q eh muito pouco provável), deve ser 
3*R*SQRT(7)/14

[]'s 
Boromir

  
  -Mensagem original-De: 
  [EMAIL PROTECTED] [mailto:[EMAIL PROTECTED] 
  Em nome de Daniel PiniEnviada em: sábado, 31 de maio de 2003 
  14:26Para: [EMAIL PROTECTED]Assunto: [obm-l] ajuda 
  geometria
  Considere um triângulo equilátero ABC, inscrito em um 
  circulo de raio R. Os pontos M e N são, respectivamente, os pontos médios do 
  arco menor AC e do segmento BC. Se a reta MN também intercepta a 
  circunferencia desse circulo no ponto P, P diferente de M, então NP 
  mede?
  


Re: [obm-l] outras questões

2004-07-18 Por tôpico Marcos Paulo
At 23:54 17/7/2004, you wrote:
1)Considere o conjunto:
s={(a,b) pertente N xN | a+b=18}
A soma de todos os valores  da forma 18!/a!b! é
a)8^6 b)9! c)9^6 d)12^6 e)12!
Essa soma será igual a C18,0 + C18, 1 + ... + C18,18 = 2^18 = (2^3)^6 = 8^6 (A)

2)A soma dos fatoriais das raízes da equação:
x^4-8x^3+19x^2-12x=0 é:
a)12  b)31 c)32 d)33 e)34
Como 0 e 1 são raízes e o oplinomio só tem 4 raizes, as outras só podem ser 
3 e 4 (se fosse 2 não teria opção)  0! + 1! + 3! + 4! = 32


3)A área do polígono, situado no primeiro quadrante , que é delimitado 
pelos eixos coordenados e pelo conjunto :
{(x,y) pertence aos R² : 3x²+2y² + 5xy-9x-8y + 6 =0}
é igual a:

a)1,5 b)2,5 c)3,0 d)3,5 e)4,0
2(x+y)^2 + x^2 + xy -9x -8y + 6
2(x+y)^2 + x^2 + xy - x - 8x - 8y + 8 - 2
2(x+y)^2 + x(x + y - 1) - 8(x + y - 1) - 2
2[(x +y)^2 - 1] + (x + y - 1)(x - 8)
2(x + y - 1)(x + y + 1) + (x + y - 1)(x - 8)
(x + y - 1)(3x + 2y - 6) = 0
deve ter um jeito mais facil de fatorar isso ...
O polígono é formado pelos pontos A(1, 0) B(2, 0), C(0, 3) e D(1, 0)
a área dá 2,5

abços
Junior

[]'s MP 

=
Instruções para entrar na lista, sair da lista e usar a lista em
http://www.mat.puc-rio.br/~nicolau/olimp/obm-l.html
=


Re: [obm-l] Escola Naval 2004

2004-07-18 Por tôpico Marcos Paulo
At 21:14 18/7/2004, you wrote:
(sem as setinhas de vetor):
U = (2,1,-3)
P= (3,-1,0)
Seja V=(a,b,c) e W=(d,e,f)
W e V são perpendiculares -  ad + be + cf = 0 (produto escalar=0)
2 = a+d - 4 = a^2 + 2ad + d^2
1 =  b^2 + 2be + e^2
 9 = c^2 +2cf + f^2
- 14 =  a^2 + b^2 + c^2 + d^2 + e^2 + f^2 + 2(ad + be + cf) = a^2 + b^2 + 
c^2 + d^2 + e^2
V - W = (a - d, b - e, c - f) -  |V - W| = sqrt2(a^2 + b^2 + c^2 + d^2 + 
e^2 + f^2 - 2(ad + be + cf) =
sqrt2(a^2 + b^2 + c^2 + d^2 + e^2 + f^2) = sqrt2(14) (letra B)
Avisem se tiver algo errado..
[]´s
Igor
Suas contas estão certas.
Mas se dois vetores (como v e w) são perpendiculares, então a norma da sua 
soma é igual a norma da sua diferença e portanto bastaria fazer |u| = 
SQRT(2^2 + 1^1 + (-3)^2) = SQRT(14)
[]'s MP


- Original Message -
From: mailto:[EMAIL PROTECTED]João Vitor
To: mailto:[EMAIL PROTECTED][EMAIL PROTECTED]
Sent: Sunday, July 18, 2004 8:12 PM
Subject: Re: [obm-l] Escola Naval 2004
Essa é de Vetores
----  -   --  - 
- -   -  -
Sabendo q:   U  = 2i +  j  -  3k  ;   U = V + W   onde V  é paralelo 
aP  =  3i   - j e   W  é

   -   -  -
perpendicular a P ; Podemos Afirmar q|V - W|   é:
A) Sqrt(19)/2
B) Sqrt(14)
C) Sqrt(27)/4
D) Sqrt(20)
E) Sqrt(53)/2
Essa caiu ano passado na Escola Naval!

João Vitor, Fortaleza - CE
 - Original Message -
From: mailto:[EMAIL PROTECTED]Robÿe9rio Alves
To: mailto:[EMAIL PROTECTED][EMAIL PROTECTED]
Sent: Saturday, July 17, 2004 8:03 PM
Subject: [obm-l] Probleminha legal, como resolver ?
Sabendo que um balaio de ovo foi dividido entre três pessoas. O primeiro 
ficou com a metade da quantidade de ovos mais meio ovo. O segundo ficou 
com a metade do que sobrou mais um muio. Por conseguinte, o último com a 
metade do que sobrou mais um meio. Pergunta - se
a) Quantos ovos ( inteiros ) há no balaio ?
b) Quantos ovos ficou a primeira pessoa ?
c) Quantos ovos ficou a segunda ?
d) Quantos ovos ficou a terceira ?

__
Do You Yahoo!?
Tired of spam? Yahoo! Mail has the best spam protection around
http://mail.yahoo.com

=
Instruções para entrar na lista, sair da lista e usar a lista em
http://www.mat.puc-rio.br/~nicolau/olimp/obm-l.html
=


Re: [obm-l] Questões estranhas

2004-08-03 Por tôpico Marcos Paulo
At 21:21 2/8/2004, you wrote:
Alguém poderia me dar uma ajuda nisso?
1 - Sabendo-se que a equação x^2*(x + 13) - 6x*(x^2 + 2) + 4 = 0 pode ser
escrita como o produto de dois binômios do primeiro grau, a soma de duas das
suas raízes distintas é igual a:
Resp.: 3
Essa questão vc copiou errada a pergunta correta é:
Sabendo que x^2 (x^2 + 13) - 6x(x^2 + 2) + 4 = 0 pode ser escrita como o 
produto de binomios do primeiro grau (e não de DOIS binomios)..

Note que o termo independente de x vale 4 e, portanto se houver alguma raiz 
inteira essa será um dos divisores de 4, ou seja, +1, -1, +2, -2, +4, -4. 
Basta testar 1 e 2 e vc verá que são raízes.

2 - O valor numérico da expressão 120x^4 + 10k^2 + 8, sendo k um natural, é
o quadrado de um número natural para:
Resp.: Nenhum valor de k

note que 120k^4 tem digito das unidades ZERO, asssim como 10k^2. POrtanto o 
dígito das unidades da expressão inteira será 8 e não existe quadrado 
perfeito que termine em 8, portanto independente do valor inteiro de k, a 
expressão nunca será um quadrado perfeito.

Quem quiser ver a prova inteira do colégio naval pode entrar no endereço 
www.cursoriachuelo.com.br/cn2004.htm neste endereço está o gabarito extra 
oficial e clicando nas questões da prova azul abre uma janelinha com a 
questão correspondente (enunciado + opções).
Se tiver um tempinho eu vou colocar em breve a prova com soluções comentadas.

[]'s Marcos Paulo 

--
Mensagens enviadas estão livres de vírus.
Verificado por AVG Anti-Vírus (http://www.avgbrasil.com.br).
Versão: 7.0.262 / Banco de dados de Vírus: 264.2.0 – Data de Lançamento: 2/8/2004

=
Instruções para entrar na lista, sair da lista e usar a lista em
http://www.mat.puc-rio.br/~nicolau/olimp/obm-l.html
=


Re: [obm-l] Re: [obm-l] Questões estranhas

2004-08-03 Por tôpico Marcos Paulo
Oi Paulo,
eu discordo da estranheza da prova. Achei que a prova foi bastante 
interessante ressucitando temas interessantes que estavam meio que às 
traças como o retângulo áureo (questão 7); o eixo radical (questão 17), a 
fórmula de transformação de radicais duplos em soma de radicais simples 
(questão 2). O produto notável pedido na questão 1 aparece em quase todo 
livro de oitava série (mesmo os piores) e eu não conheço outra 
justificativa (ou uma melhor) para que uma divisão entre inteiros resulte 
numa dízima periódica a não ser o fato de que exista uma quantidfade finita 
de restos possíveis na divisão, enquanto o processo (o algoritmo da 
divisão) pode ser repetido infinitamente.
Talvez a opinião dos outros membros da lista fosse interessante nessa questão.

[]'s MP
P.S. Os números das questões que eu citei são referentes à prova azul.
At 14:12 3/8/2004, you wrote:
Essa prova do CN está esquisita mesmo. Vocês viram as questões 1 e 16?
No caso da 16, a resposta certa é a única que faz algum sentido, mas dá a
entender que toda seq. com uma quantidade limitada de valores é periódica!
Essas provas do CN já não foram melhores?
Paulo

--
Mensagens enviadas estão livres de vírus.
Verificado por AVG Anti-Vírus (http://www.avgbrasil.com.br).
Versão: 7.0.262 / Banco de dados de Vírus: 264.2.0 – Data de Lançamento: 2/8/2004

=
Instruções para entrar na lista, sair da lista e usar a lista em
http://www.mat.puc-rio.br/~nicolau/olimp/obm-l.html
=


Re[3]: [obm-l] [obm-l] Questões estranhas

2004-08-03 Por tôpico Marcos Paulo
Concordo totalmente. Para economizar uma palavra eles estragaram o enunciado.
A questão diz:
Um número natural N tem 2005 divisores positivos. Qual o número de bases distintas de 
sua decomposição em fatores primos.
Dessa forma o número pode ser 
N = (primo)^2004 que tem 2005 divisores positivos e uma base só
ou
N = (p1)^4 * (p2)^400 que tambem tem 2005 divisores positivos e tem duas bases 
diferentes.

Se tivessem acrescentado a palavra máximo (ou mínimo) do lado da palavra  número...

[]'s MP


=
De:Augusto Cesar de Oliveira Morgado [EMAIL PROTECTED]
Para:[EMAIL PROTECTED]
Assunto:Re: [obm-l] Re: [obm-l] Questões estranhas

Na questao 18, o numero procurado pode ser 1 ou
2.


==
Mensagem  enviada  pelo  CIP  WebMAIL  - Nova
Geração - v. 2.1
CentroIn Internet Provider 
http://www.centroin.com.br
Tel: (21) 2542-4849, (21) 2295-3331Fax:
(21) 2295-2978
Empresa 100% Brasileira - Desde 1992 prestando
servicos online


-- Original Message ---
From: Marcos Paulo [EMAIL PROTECTED]
To: [EMAIL PROTECTED]
Sent: Tue, 03 Aug 2004 16:16:31 -0300
Subject: Re: [obm-l] Re: [obm-l] Questões
estranhas

 Oi Paulo,
 eu discordo da estranheza da prova. Achei que
a prova foi bastante 
 interessante ressucitando temas interessantes
que estavam meio que 
 às traças como o retângulo áureo (questão 7);
o eixo radical 
 (questão 17), a fórmula de transformação de
radicais duplos em soma 
 de radicais simples 
 (questão 2). O produto notável pedido na
questão 1 aparece em quase 
 todo livro de oitava série (mesmo os piores) e
eu não conheço outra 
 justificativa (ou uma melhor) para que uma
divisão entre inteiros 
 resulte numa dízima periódica a não ser o fato
de que exista uma 
 quantidfade finita de restos possíveis na
divisão, enquanto o 
 processo (o algoritmo da divisão) pode ser
repetido infinitamente. 
 Talvez a opinião dos outros membros da lista
fosse interessante 
 nessa questão.
 
 []'s MP
 
 P.S. Os números das questões que eu citei são
referentes à prova azul.
 
 At 14:12 3/8/2004, you wrote:
 
 Essa prova do CN está esquisita mesmo. Vocês
viram as questões 1 e 16?
 
 No caso da 16, a resposta certa é a única que
faz algum sentido, mas dá a
 entender que toda seq. com uma quantidade
limitada de valores é periódica!
 
 Essas provas do CN já não foram melhores?
 
 Paulo
 
 -- 
 Mensagens enviadas estão livres de vírus.
 Verificado por AVG Anti-Vírus
(http://www.avgbrasil.com.br).
 Versão: 7.0.262 / Banco de dados de Vírus:
264.2.0 – Data de 
 Lançamento: 2/8/2004
 


=
 Instruções para entrar na lista, sair da lista
e usar a lista em

http://www.mat.puc-rio.br/~nicolau/olimp/obm-l.h
tml


=
--- End of Original Message ---


=
Instruções para entrar na lista, sair da lista e
usar a lista em
http://www.mat.puc-rio.br/~nicolau/olimp/obm-l.h
tml

=







=
Instruções para entrar na lista, sair da lista e usar a lista em
http://www.mat.puc-rio.br/~nicolau/olimp/obm-l.html
=


Re[2]: [obm-l] CN 2004

2004-08-04 Por tôpico Marcos Paulo
Outra maneira bastante razoavel seria lembrar que
SQRT(A + SQRT(B)), fazendo C = A²-B, será:

SQRT((A+C)/2) + SQRT((A-C)/2).

Aplicando 2 vezes isso vc encontra o resultado:

SQRT(SQRT(49 + sqrt(2400))) - C = 1
SQRT(SQRT(49 + sqrt(2400))) = SQRT(SQRT(25) + SQRT(24)) =
= SQRT(5 + SQRT(24) - C' = 1 novamente
SQRT(5 + SQRT(24) = SQRT(3) + SQRT(2) = 3,1463 (aproximadamente)

[]'s MP






=
Instruções para entrar na lista, sair da lista e usar a lista em
http://www.mat.puc-rio.br/~nicolau/olimp/obm-l.html
=


Re: [obm-l] Duvidas

2004-08-04 Por tôpico Marcos Paulo
At 22:26 4/8/2004, you wrote:
Olá !
x=7a+5=13b+9=7a-13b=5; a,b,c naturais
OPa ..
7a - 13b = 4

Dai a=(5+13b)/7
a = (4 + 13b)/7

Como a é suposto inteiro = b=7c+5
b = 7c + 4

Daí, substituindo na primeira relação, temos que:
x=13(7c+5)+9=91c+65
x = 13(7c + 4) + 9 = 91c + 61

Alternativa c)
Alternativa b
Eu particularmente acho mais legal resolver esse tipo de questão 
encontrando uma solução particular:

quando vc chega a
7a - 13b = 4
note que a = 8 e b = 4 é uma solução possível.
Daí N = 7*8 + 5 é um possível valor para N e se esse valor particular deixa 
resto 61 todos os outros valores de N tb deixarão resto 61.
Há diversas técnicas de encontrar solução particular (por exemplo usar o 
algoritmo de euclides para expressar o mdc entre 7 e 13 como uma combinação 
linear deles e depois multiplicar por 4..) na maioria dos exercicios a 
solução particular pode ser claculada por tentativas.
Há alguns artigos na Eureka e na RPM sobre esse tipo de equação dita 
diofantina.

[]'s MP

Até mais.


 Eu acho que está faltando uma alternativa nesta
questão a letra d como
 sendo 63 , voces não acham?



Como se faz esta questao sem usar congruencias?
 
 Um numero natural ao ser dividido por 7 deixa
resto
  5 e , ao ser dividido por 13 , deixa resto resto 9.
O
  resto da divisão desse número por 91 eh igual a:
 
  a0 45 b0 61c) 65   e)75
 
 
 
agradeço desde de já.
 
 

__
  Acabe com aquelas janelinhas que pulam na sua tela.
  AntiPop-up UOL - É grátis!
  http://antipopup.uol.com.br/
 
 
 
 

=
  Instruções para entrar na lista, sair da lista e
usar a lista em
  http://www.mat.puc-rio.br/~nicolau/olimp/obm-l.html
 

=
 




=
 Instruções para entrar na lista, sair da lista e usar
a lista em
 http://www.mat.puc-rio.br/~nicolau/olimp/obm-l.html


=

Atenciosamente,
Osvaldo Mello Sponquiado
2º ano em Engenharia Elétrica
UNESP - Ilha Solteira
__
Acabe com aquelas janelinhas que pulam na sua tela.
AntiPop-up UOL - É grátis!
http://antipopup.uol.com.br/

=
Instruções para entrar na lista, sair da lista e usar a lista em
http://www.mat.puc-rio.br/~nicolau/olimp/obm-l.html
=

--
Este e-mail está livre de vírus.
Verificado por AVG Anti-Vírus (http://www.avgbrasil.com.br).
Versão: 7.0.262 / Banco de dados de Vírus: 264.2.0 ­ Data de Lançamento: 
2/8/2004

--
Mensagens enviadas estão livres de vírus.
Verificado por AVG Anti-Vírus (http://www.avgbrasil.com.br).
Versão: 7.0.262 / Banco de dados de Vírus: 264.3.0 – Data de Lançamento: 4/8/2004

=
Instruções para entrar na lista, sair da lista e usar a lista em
http://www.mat.puc-rio.br/~nicolau/olimp/obm-l.html
=


Re: [obm-l] Morgado, Guilherme e a todos amigos

2004-08-09 Por tôpico Marcos Paulo




=
De:nilton rr [EMAIL PROTECTED]
Para:[EMAIL PROTECTED]
Assunto:[obm-l] Morgado, Guilherme e a todos amigos





Por favor companheiros preciso dessas respostas
até quarta feira dia da minha aula, agradeço
antecipadamente.
1) Uma pastelaria vende pastéis de palmito,
carne, queijo e espinafre. de quantas maneiras
uma pessoa pode comprar 8 pastéis sendo pelo
menos 2 de queijo?
Seja p o número de pastéis de palmito, y o número de pastéis de carne, z o número de 
pastéis de queijo e w o número de pastéis de espinafre.
p + y + z + w = 8. Ainda devemos ter p maior que 2, portanto faremos p = x + 2 e dessa 
forma mesmo q x seja zero p será pelo menos 2.
A pergunta agora será: Quantas soluções naturais tem a equação
x + y + z + w = 6?

Para repartir 6 unidades (u) em 4 grupos são necessários 3 separadores (s)
Por exemplo:
s u u s u u u s u seria o equivalenbte a x = 0, y = 2, z = 3 e w = 1

Cada permutação dessas 9 letras será uma resposta nova, portanto a minha resposta será 
igual ao número de permutações da palavra acima (formada por s e u)
Resposta:
9!/(3!*6!)



2)Uma livraria vai doar 25 livros iguais a 5
bibliotecas municipais. Cada biblioteca deve
receber pelo menos 3 livros. qual o nr de
maneiras distintas que esta livraria poderá
repartir os livros dessa doação?

Repetindo o processo acima vc tem:
X1+X2+X3+X4+X5 = 25 e faça X1 = 2+A1, ... X5 = 3+A5
resolva igual ao primeiro exercício.

3) quantas são as soluções inteiras não
negativas de x1+x2+x3+x4+x5+x6+x7 =15 nas quais
exatamente duas incógnitas são nulas?

Aqui vc tem q escolher 2 entre 7 para serem nulas C7, 2
Vão sobrar 5 incógnitas: A1, A2, ...,A5 substitua cada uma delas por B1+1, B2+1, ..B5 
+ 1 e resolva igual ao primeiro exercicio encontrando um resultardo R a resosta será 
R* C7,2

no livro Do Morgado (Que pra alegria de muita gente está numa edição nova com os 
exercícios resolvidos) Há uma parte da teoria bastante util em exercicios como esse 
das combinações completas (ou com repetição). Resumidamente, se vc quer escolher p 
objetos dentre n disponíveis sendo que nesses p vc pode repetir a sua escolha o número 
de maneiras de faze-lo é CRn,p. A relação entre o núemro de combinações completas e o 
número de combinações simples (onde vc deve necesariamente fazer escolhas de objetos 
distintos) é:
CRn,p = C(n+p-1),p

Espero não ter confundido tudo..

[]'s MP





=
Instruções para entrar na lista, sair da lista e usar a lista em
http://www.mat.puc-rio.br/~nicolau/olimp/obm-l.html
=


Re: [obm-l] GEOMETRIA E IMAGINAÇÃO!

2004-08-09 Por tôpico Marcos Paulo




=
De:[EMAIL PROTECTED]
Para:[EMAIL PROTECTED]
Assunto:[obm-l] GEOMETRIA E IMAGINAÇÃO!

PASMEM! Este problema de geometria, proposto
numa prova para mais de um milhão
de alunos, teve somente um único acertador,
Daniel Lowen, de 17 anos da Escola
Cocoa Beach

Seja ABCDE uma pirâmide de base quadrada, cujas
faces laterais são triângulos
equiláteros; e seja FGHI um tetaedro regular
cujas faces sejam (triângulos
equiláteros) congruentes às faces laterais da
pirâmide. Suponhamos que se
juntem os sólidos de maneira que a face ADE da
pirâmide coincida com a face GIH
do tetaedro, o resultado sendo o poliedro
ABCDEF. Quantas faces tem este
poliedro?
Há Uma face quadrangular e 6 faces trîangulares.



 
   
   (Educational Testing Service-EUA)

NOTA: Meus amigos, sem nenhum exagero, este é um
problema fascinante. Nunca vi
nada igual. (CAMPEÃO!).



__
WebMail UNIFOR - http://www.unifor.br.

=
Instruções para entrar na lista, sair da lista e
usar a lista em
http://www.mat.puc-rio.br/~nicolau/olimp/obm-l.h
tml

=







=
Instruções para entrar na lista, sair da lista e usar a lista em
http://www.mat.puc-rio.br/~nicolau/olimp/obm-l.html
=


Re[2]: [obm-l] PARADOXO DA UNIDADE!

2004-08-09 Por tôpico Marcos Paulo




=
De:Qwert Smith [EMAIL PROTECTED]
Para:[EMAIL PROTECTED]
Assunto:RE: [obm-l] PARADOXO DA UNIDADE!

-1 = i . i ...OK
= (-1)^(1/2) . (-1)^(1/2) ... OK
= (-1) . (-1)^(1/2) ...epa! = ( i ) .(-1)^(1/2)
= (1)^(1/2) = 1

Eu acho q não dá pra substituir i por (-1)^1/2 porque (-1)^1/2 = 1^1/2 cis (pi/2 + 
kpi) = i ou = -i por isso é bom tomarmos cuidado quando formos definir i acho mais 
interessante definir i^2 = -1 do que i = SQRT(-1)

A propósito, por que o círculo trigonométrico
tem raio igual a 1?
preguica... se o raio fosse h... ao invez de
sen(x) = y
teriamos ki escrever sen(x) = y/h
Acho que não é preguiça não .. a resposta talvez seja: Porque é permitido. Quando 
vamos estudar um problema qualquer é permitido que escolhamos a unidade de 
comprimento. No caso de estudar o círculo é permitido tomarmos o raio como unidade de 
comprimento. Obviamente essa é uma boa escolha porque facilita algumas contas. mas o 
principal é deixar claro que não há roubo em escolher unidades convenientes no 
estudo de uma situação.

Por que todo número elevado à zero é igual a 1?
a^n/a^n = a^(n-n) = a^0 = 1

Qualquer número elevado a zero é um por definição. Definimos dessa forma se queremos 
que valha a propriedade q o Qwert citou.


Por que -1 multiplicado por -1 é igual a 1?
Porque n . -1 = - n para qualquer n, -(-1)=1

Esse assunto (talvez os outros tb) já estiverma na berlinda nessa lista há um tempo 
atras .. talvez fose uma boa olhar nos arquivos..

Sendo a0 por que, quando n cresce
indefinidamente, (a)^(1/n) tende a 1???

porque 1/n tende a 0 e como ja vimos a^0 eh 1

...
Por favor nao deixem que a alta formalidade das
respostas em conjunto com a
aplicacao impecavel de teoria da matematica
iniba vcs de responderem tambem.
Um dia vcs tb chegam la.


_
Express yourself instantly with MSN Messenger!
Download today - it's FREE! 
hthttp://messenger.msn.click-url.com/go/onm00200
471ave/direct/01/


=
Instruções para entrar na lista, sair da lista e
usar a lista em
http://www.mat.puc-rio.br/~nicolau/olimp/obm-l.h
tml

=







=
Instruções para entrar na lista, sair da lista e usar a lista em
http://www.mat.puc-rio.br/~nicolau/olimp/obm-l.html
=


Re: [obm-l] RES: [obm-l] GEOMETRIA E IMAGINAÇÃO!

2004-08-09 Por tôpico Marcos Paulo
É verdade  .. quando o Guilherme mandou a resposta eu fui fazer as contas 
tb ...
Não me admiro que tanta gente tenha errado.

[]'s MP
At 00:02 10/8/2004, you wrote:
Marcos, acho que você se equivocou na resposta.
Realmente Jorge, o problema é interessantíssimo e nunca tinha me deparado
com algo similar.
Se fizerem um esboço do poliedro resultante vão ver que existe a junção de
dois ângulos poliédricos.
Fazendo os pontos A=G, D=I, E=H.
Vamos encontrar o valor destes ângulos.
No Tetraedro:
l^2 = 2(l*sqrt(3)/2)^2 - 2[(l*sqrt(3)/2)^2]* cos(T)
Resolvendo temos que cos(T) = 1/3
Na Pirâmide:
[L*sqrt(2)]^2 = 2(l*sqrt(3)/2)^2 - 2[(l*sqrt(3)/2)^2]* cos(P)
Resolvendo temos que cos(P) = -1/3
Com isso descobrimos que estes ângulos são suplementares, e a junção deles
forma um plano perfeito!
Isto ocorre com a junção de duas faces do tetraedro com a pirâmide, e o
sólido resultante possui duas faces triangulares e três faces
quadrangulares.
Belíssima questão de Geometria Jorge! Se tiver mais dessas mande! =)
Um abraço, Douglas Ribeiro Silva
-Mensagem original-
De: [EMAIL PROTECTED] [mailto:[EMAIL PROTECTED] Em nome
de Marcos Paulo
Enviada em: segunda-feira, 9 de agosto de 2004 20:50
Para: [EMAIL PROTECTED]
Assunto: Re: [obm-l] GEOMETRIA E IMAGINAÇÃO!


=
De:[EMAIL PROTECTED]
Para:[EMAIL PROTECTED]
Assunto:[obm-l] GEOMETRIA E IMAGINAÇÃO!

PASMEM! Este problema de geometria, proposto
numa prova para mais de um milhão
de alunos, teve somente um único acertador,
Daniel Lowen, de 17 anos da Escola
Cocoa Beach

Seja ABCDE uma pirâmide de base quadrada, cujas
faces laterais são triângulos
equiláteros; e seja FGHI um tetaedro regular
cujas faces sejam (triângulos
equiláteros) congruentes às faces laterais da
pirâmide. Suponhamos que se
juntem os sólidos de maneira que a face ADE da
pirâmide coincida com a face GIH
do tetaedro, o resultado sendo o poliedro
ABCDEF. Quantas faces tem este
poliedro?
Há Uma face quadrangular e 6 faces trîangulares.


   (Educational Testing Service-EUA)

NOTA: Meus amigos, sem nenhum exagero, este é um
problema fascinante. Nunca vi
nada igual. (CAMPEÃO!).



__
WebMail UNIFOR - http://www.unifor.br.

=
Instruções para entrar na lista, sair da lista e
usar a lista em
http://www.mat.puc-rio.br/~nicolau/olimp/obm-l.h
tml

=



=
Instruções para entrar na lista, sair da lista e usar a lista em
http://www.mat.puc-rio.br/~nicolau/olimp/obm-l.html
=
=
Instruções para entrar na lista, sair da lista e usar a lista em
http://www.mat.puc-rio.br/~nicolau/olimp/obm-l.html
=

--
Este e-mail está livre de vírus.
Verificado por AVG Anti-Vírus (http://www.avgbrasil.com.br).
Versão: 7.0.262 / Banco de dados de Vírus: 264.5.0 ­ Data de Lançamento: 
9/8/2004

--
Mensagens enviadas estão livres de vírus.
Verificado por AVG Anti-Vírus (http://www.avgbrasil.com.br).
Versão: 7.0.262 / Banco de dados de Vírus: 264.5.0 – Data de Lançamento: 9/8/2004

=
Instruções para entrar na lista, sair da lista e usar a lista em
http://www.mat.puc-rio.br/~nicolau/olimp/obm-l.html
=


Re: [obm-l] curso de aperfeiçoamento dos professores

2004-08-10 Por tôpico Marcos Paulo
Tente em
http://strato.visgraf.impa.br/capem_jul2004.html
At 18:32 10/8/2004, you wrote:
Deve ser http://milenio.impa.br/http://milenio.impa.br Link Teaching, 
Popularization, Olympiads)

Thiago Ferraiol [EMAIL PROTECTED] wrote:
Olá pessoal...
Alguem saberia me responder se existe algum lugar onde posso baixar os 
vídeos referentes as palestras do curso de aperfeiçoamento dos professores 
de matemática, promovidos pelo Impa???

Eu baixei alguns videos no ano passado mas perdi o link!!! Será que alguém 
pode me ajudar!!??

Obrigado...
[]'s Thiago Ferraiol
http://br.rd.yahoo.com/mail/taglines/*http://br.yahoo.com/mail/taglines/?http://br.acesso.yahoo.com/Yahoo! 
Acesso Grátis - navegue de graça com conexão de qualidade!

http://br.rd.yahoo.com/mail/taglines/*http://br.yahoo.com/mail/taglines/?http://br.acesso.yahoo.com/Yahoo! 
Acesso Grátis - navegue de graça com conexão de qualidade!

Este e-mail está livre de vírus.
Verificado por AVG Anti-Vírus (http://www.avgbrasil.com.br).
Versão: 7.0.262 / Banco de dados de Vírus: 264.5.0 ­ Data de Lançamento: 
9/8/2004

--
Mensagens enviadas estão livres de vírus.
Verificado por AVG Anti-Vírus (http://www.avgbrasil.com.br).
Versão: 7.0.262 / Banco de dados de Vírus: 264.5.0 – Data de Lançamento: 9/8/2004

=
Instruções para entrar na lista, sair da lista e usar a lista em
http://www.mat.puc-rio.br/~nicolau/olimp/obm-l.html
=


Re: [obm-l] matematica-epcar

2004-08-17 Por tôpico Marcos Paulo
Quando ele fez c = 10 ele não pode é usar a relação que vc usou para o 
calculo dos divisores.
Y = 2^1 * 3^2 *10^2 (não está fatorado em fatores primos)
Y = 2^3 * 3^2 * 5^2 (agora sim e aí vc pode usar a sua fórmula para 
encontrar o número de divisores)

Embora eu acredite que o gabarito oficial deve dar como resposta a letra B 
eu concordo que (como está escrito o enunciado aqui) caberia tambem a letra 
C como resposta.

Somente terei acesso à prova do primeiro ano amanhã, para colocar o 
gabarito + as questões no site do curso em que trabalho. HOje eu só vi a 
prova do terceiro ano (que parece q tem uma questão com problemas no 
gráfico (uma função definida num intervalo mas que no gráfico apresenta 
descontinuidade de modo que um elemento do domínio não tem associado no 
contradominio)

[]'s MP

/cara você fez totalmente correta a questão.
Repara que se você adotar a= 1 b=2 c= 10, o número será divisível por N.
Porém não terá 36 divisores, repare:
(a+1)(b+1)(2+1) =36
2 x 3 x 3
18=36
logo você não poderia trabalhar com esse números!
parabéns pelo seu desempenho, espero fazer algo pareciso no sábado
abços
Junior
---
Em um e-mail de 17/8/2004 17:42:19 Hora oficial do Brasil, 
[EMAIL PROTECTED] escreveu:

ops
ate que fim ,parece que acertei 28 questoes das 30 de
acordo com os gabaritos extras oficiais
Venho pedir auxilio sobre a formulaçao de uma questao
(epcar)
VERSAO:C
12.   O NUMERO Y=2^(A).3^(B).C^(2)é divisor de
N=15.20.6.sabendo-se que Y admite extamente 36
divisores,é correto afimar que
A)ab=c   C)abc
B)a+b=c  D)a-b=-1
==
eu resolvi da seguinte maneira
N=2^(3).3^(2).5^(2)
considerando que o numero 'C' seja um numero primo
vem que 'c' é igual a 5 e
vem que (a+1).(b+1).(2+1)=36
a+b+ab=11 o que nos convem a=3,b=2
entao a resposta certa é a letra (b) a+b=c
===
MAS O ENUCIADO NAO MENCIONA QUE (c) TEM QUE SER UM
NUMERO PRIMO
ENTAO PODERIAMOS CONSIDERA QUE
A=1,B=2 E C=10
TENDO DE ACORDO COM O ENUCIADO 36 DIVISORES E 'Y'
IGUAL A 'N'
 ENTAO TERIA COMO RESPOSTA A LETRA (C) E (D)

ATENCIOSAMENTE
LEANDRO GERALDO DA COSTA 'EU SOU BOM'


Este e-mail está livre de vírus.
Verificado por AVG Anti-Vírus (http://www.avgbrasil.com.br).
Versão: 7.0.262 / Banco de dados de Vírus: 264.6.3 ­ Data de Lançamento: 
16/8/2004

--
Mensagens enviadas estão livres de vírus.
Verificado por AVG Anti-Vírus (http://www.avgbrasil.com.br).
Versão: 7.0.262 / Banco de dados de Vírus: 264.6.3 – Data de Lançamento: 16/8/2004

=
Instruções para entrar na lista, sair da lista e usar a lista em
http://www.mat.puc-rio.br/~nicolau/olimp/obm-l.html
=


Re: [obm-l] matematica-epcar

2004-08-18 Por tôpico Marcos Paulo
Hoje eu tive acesso às provas para examinar com mais calma.
A questão exatamente como está na prova tem o seguinte enunciado:
O número y = 2^a x 3^b x c^2 é divisor de N = 15 x 20 x 6. Sabendo-se que y 
admite exatamente 36 divisores, é correto afirmar que:
obs.: Considere x o sinal de multiplicação
a) ab = c
b) a + b = c
c) a  b  c
d) a - b = -1

(acho q a mensagem inical tinha o enuncado correto)
Bem:
1) y tem 36 divisores e não 36 divisores naturais (ou positivos). Há 18 
divisores positivos de y.
2) c pode ou não ser um número primo
3) N = 2^3 * 3^2 * 5^2
Resolvendo:
1º caso: c é primo e, portanto, igual a 2, 3 ou 5
p/ c = 2:

y = 2^(a + 2) x 3^b
(a + 3)(b + 1) = 18
18 admite as seguintes fatorações:
1 * 18; 2 * 9; 3 * 6
Somente podemos considerar  3 * 6, porque as outras darão a ou b maiores do 
que seus valores máximos
1) a + 3 = 6 - a = 3 e b + 1 = 3 - b = 2 (não há opções que sejam 
satisfeitas para esse resultado)
2) a + 3 = 3  - a = 0 e b + 1 = 6 - b = 5 (não serve pois b é no máximo 2)

p/ c = 3
y = 2^a x 3^(b+2)
(a+1)(b+3)=18
1)  a = 2 e b = 3 (opção certa letra d)
2) a = 5 e b = 0 (não serve)
p/ c = 5
y = 2^a x 3^b x 5^2
(a+1)(b+1)*3 = 18 - (a+1)(b+1) = 6 = 1*6 = 2*3 (1 * 6 não serve)
1) (a+1) = 2 - a = 1 e (b+1) = 3 - b = 2 (letra d ou letra c)
2) a = 2 e b = 1 (não há opção certa)
2º Caso: c não e primo (c = 6 ou c = 10 ou c = 15)
p/ c = 6
y = 2^(a+2) x 3^(b+2)
(a+3)(b+3) = 18
a = 0 e b = 3 (letra c)
b = 0 e a = 3 (sem resposta)
p/ c = 10
y = 2^(a+2)* 3^b * 5^2
(a+3)(b+1) = 6
a + 3 = 6 - a = 3 e b + 1 = 1 - b = 0 (sem resposta)
a+ 3 = 3 - a = 0 e  b + 1 = 2 - b = 1 (letras c e d)
p/ c = 15
y = 2^a * 3^(b+2) * 5^2
(a+1)(b+3) = 6
a+1 = 2 - a = 1 e b + 3 = 3 - b = 0 (sem opções)
Resumindo: Não é correto afirmar nada sem fazer conjecturas subjetivas 
antes, ou ainda essa questão foi uma lambança e tanto!!

[]'s MP

--
Mensagens enviadas estão livres de vírus.
Verificado por AVG Anti-Vírus (http://www.avgbrasil.com.br).
Versão: 7.0.262 / Banco de dados de Vírus: 264.6.3 – Data de Lançamento: 16/8/2004

=
Instruções para entrar na lista, sair da lista e usar a lista em
http://www.mat.puc-rio.br/~nicolau/olimp/obm-l.html
=


Re: [obm-l] RE: [obm-l] Demonstração Teorema Laplace...

2004-08-19 Por tôpico Marcos Paulo
Seja A = ([a1, b1, c1], [a2, b2, c2], [a3, b3, c3]) uma matriz de ordem 3.
detA = a1b2c3 + a2b3c1 + a3b1c2 - a3b2c1 - a2b1c3 - a1b3c2

detA = a1b2c3 - a1b3c2 + a2b3c1 - a2b1c3 + a3b1c2 - a3b2c1

detA = a1(b2c3 - b3c2) - a2(b1c3 - b3c1) + a3(b1c2 - b2c1)
Seja A1 = b2c3 - b3c2 (menor de a1)
A2 = b1c3 - b3c1 (menor de a2)
A3 = b1c2 - b2c1 (menor de a3)

detA = a1A1 - a2A2 + a3A3

Note que o menor de um elemento é igual ao determinantes da matriz obtida quando 
suprimimos a linha e a coluna do elemento dado.

Se vc tiver demonstrado as propriedades dos determinantes previamente vc mostra que 
isso vale para qualquer fila (linha ou coluna) da matriz, desde que feito um ajuste 
nos sinais (que seão positivos ou negativos dependendo da soma do nº da linha com o nº 
da coluna q o elemento ocupa)

Talvez essa não seja uma demonstração generalizada (para matrizes de ordem n) mas já 
quebra um galhão no ensino médio pela sua simplicidade.

Obs.: Essa demonstração consta no livro A Matemática do Ensino Médio Vol.3 que faz 
parte da Coleção do Professor de Matemática publicada pela SBM - na verdade o que está 
escrito acima é um razoavel arremedo da demonstração feita lá.

[]'s MP
=






=
Instruções para entrar na lista, sair da lista e usar a lista em
http://www.mat.puc-rio.br/~nicolau/olimp/obm-l.html
=


Re[3]: [obm-l] [obm-l] algumas de combinat ória

2004-08-25 Por tôpico Marcos Paulo
Um polígono não é um conjunto de pontos soltos mas sim um conjuntos de pontos 
ordenados. Podemos usar a ordem das 6 primeiras letras do alfabeto para ordenar esses 
pontos. O que o Claudio fez foi dispor 6 letras em 6 lugares diferentes em uma 
circunferencia (problema das permutaçoes circulares) e considerar os polígonos ABCDEF 
e FEDCBA o mesmo polígno.
O total das permutações circulares é (6-1)! e o resultado foi metade desse numero 
(porque os polígnos q eu citei acima foram contados como distintos quando são iguais)

Acho q foi isso...

[]'s MP



=
De:Andre Silveira Ramos [EMAIL PROTECTED]
Para:[EMAIL PROTECTED]
Assunto:Re: [obm-l] Re: [obm-l] algumas de combinat ória

Claudio...
nao entendi o porque de 5!/2.

Andre


 --- Claudio Buffara
[EMAIL PROTECTED]
escreveu: 
 Na (ii), o enunciado estah realmente um pouco
 ambiguo.
 Por um lado, temos a interpretacao de que
todos os 6
 vertices devem ser
 usados.
 Nesse caso, a resposta eh 5!/2 = 60.
 
 Por outro lado, pode ser que se queira o
numero de
 triangulos,
 quadrilateros, pentagonos e hexagonos que
tenham
 estes pontos como vertices.
 Nesse caso, a resposta seria:
 Comb(6,3) + Comb(6,4)*3*2*1/2 +
Comb(6,5)*4*3*2*1/2
 + Comb(6,6)*5*4*3*2*1/2
 =
 20 + 45 + 72 + 60 = 197
 
 []s,
 Claudio.
 
 on 24.08.04 07:08, Thor at [EMAIL PROTECTED]
 wrote:
 
 ii) Dado 6 pontos sobre a circunferencia ,
podemos
 formar ( triangulos ,
 quadrilateros , pentagonos e um hexagono),
logo
 teremos
  
 Combinaçao de 6 , 3 a 3 + combinaçao de 6 , 4
a 4 +
 combinaçao de 6 , 5 a 5
 + combinaçao 6 , 6 a 6 , fazendo as contas
 20+15+6 +1=32 poligonos.
  
 Tenho que ir, vou dar minha aulinha!
  
 Espero ter ajudado.
  
Cláudio Thor.
 - Original Message -
 From: Andre Silveira Ramos
 mailto:[EMAIL PROTECTED]
 To: [EMAIL PROTECTED]
 Sent: Tuesday, August 24, 2004 1:44 AM
 Subject: [obm-l] algumas de combinatória
 
 Aí pessoal, estou com alguns problemas de
 combinatória que não estou
 conseguindo sair do lugar.
 Preciso de algumas dicas
 
 (i) Considere um conjunto P de 30 pontos do
espaço e
 P1 um subconjunto de 12
 pontos coplanares de P. Sabe-se que sempre que
4
 pontos de P são coplanares,
 então eles são pontos de P1. Quantos são os
planos
 que contém pelo menos 3
 pontos de P?
 
 (ii) Sobre uma circunferência existem 6 pontos
 distintos. Quantos polígonos,
 não necessariamente convexos, podemos
construir
 tendo por vértices esses 6
 pontos?
 
 (iii) Um bote tem 8 lugares, 4F e 4A. De
quantas
 maneiras podemos escolher
 uma tripulação para o bote se dos 31
candidatos, 10
 preferem F, 12 preferem
 A e 9 não têm preferência?
 
 (iv) Calcular a soma de todos os números de 5
 algarismos distintos formados
 com os algarismos 1, 3, 5, 7 e 9.
 
 respostas:
 (i) 3.841
 (ii) 60
 (iii) SOMATORIO (i de 0 até 4) de: C(9;k) x
 C(10;4-k) x C(21-k;4)
 (iv) 6.666.600 
 
 Obrigado...
 Abraços
 
 André
 
 
 
 Yahoo! Acesso Grátis

http://br.rd.yahoo.com/mail/taglines/*http://br
.acesso.yahoo.com/
  -
 navegue de graça com conexão de qualidade!
 
 
 
  






___
Yahoo! Acesso Grátis - navegue de graça com
conexão de qualidade! 
http://br.acesso.yahoo.com/

=
Instruções para entrar na lista, sair da lista e
usar a lista em
http://www.mat.puc-rio.br/~nicolau/olimp/obm-l.h
tml

=







=
Instruções para entrar na lista, sair da lista e usar a lista em
http://www.mat.puc-rio.br/~nicolau/olimp/obm-l.html
=


Re: [obm-l] Re: [obm-l] Essa questão é interessante ( Resolvam )

2004-08-29 Por tôpico Marcos Paulo
Acho q ele se referiu ao fato de ela poder ser respondida usando
x' + x  = - log5 = -(log10 -log2) = (log 2) +( - 1)
como essas parcelas têm produto -log2, são as raízes
[]'s MP
At 22:38 28/8/2004, you wrote:
Desculpe-me, mas o que há de interessante nessa questão?
Discriminante = (log 5)^2 + 4 log 2 = (1 - log 2)^2 + 4 log 2 = 1 + 2 log 
2 + (log 2)^2 = (1 + log 2)^2

x = [-log 5 +- (1 + log 2)]/2 = [log 2 - 1 +- (1 + log 2)]/2
x = (log 2 - 1 + 1 + log 2)/2 = log 2
ou
x = (log 2 - 1 - 1 - log 2)/2 = -1
V = {-1, log 2}
[]s,
Rafael
- Original Message -
From: mailto:[EMAIL PROTECTED]Robÿe9rio Alves
To: mailto:[EMAIL PROTECTED][EMAIL PROTECTED]
Sent: Saturday, August 28, 2004 9:11 PM
Subject: [obm-l] Essa questão é interessante ( Resolvam )
Resolva, em R, a  equação do 2º grau x^2 + x.log 5 - log 2 = 0 .

Este e-mail está livre de vírus.
Verificado por AVG Anti-Vírus (http://www.avgbrasil.com.br).
Versão: 7.0.262 / Banco de dados de Vírus: 264.7.1 ­ Data de Lançamento: 
27/8/2004

--
Mensagens enviadas estão livres de vírus.
Verificado por AVG Anti-Vírus (http://www.avgbrasil.com.br).
Versão: 7.0.262 / Banco de dados de Vírus: 264.7.1 – Data de Lançamento: 27/8/2004

=
Instruções para entrar na lista, sair da lista e usar a lista em
http://www.mat.puc-rio.br/~nicolau/olimp/obm-l.html
=


Re: [obm-l] geometria

2004-09-08 Por tôpico Marcos Paulo
[X] - área da figura X

(PC/PA)*(QA/QM)*(BM/BC)= 1 (teorema de Menelaus)
BC = 6BM - MC = 6BM - [AMC]=5S/6
[AQP]/[AMC] = (AQ/AM)*(AP/AC)=(3/4)*(2/3)
[AQP] = 0,5*[AMC]=5S/12
[AQP] = 3*[QPM]
[QPM] = 5S/36

[]'s MP




=
De:eritotutor [EMAIL PROTECTED]
Para:obm-l [EMAIL PROTECTED]
Assunto:[obm-l] geometria

Boa noite, 

Gostaria, por favor, de um auxilio na seguinte
questao:

Consideremos ABC um triangulo e  AM e BP são
cevianas 
desse triangulo, sendo M um ponto do segmento BC
e P  
um ponto do segmento AC.
Essas cevianas se interceptam num ponto Q.
Sabendo que 
a area do triangulo ABC eh S, que AP = 2PC e que
AQ = 3QM. 
Calcular o valor da area do triangulo PQM em
funçao 
area do triangulo ABC.

Obrigado
 

__
Acabe com aquelas janelinhas que pulam na sua
tela.
AntiPop-up UOL - É grátis!
http://antipopup.uol.com.br/




=
Instruções para entrar na lista, sair da lista e
usar a lista em
http://www.mat.puc-rio.br/~nicolau/olimp/obm-l.h
tml

=







=
Instruções para entrar na lista, sair da lista e usar a lista em
http://www.mat.puc-rio.br/~nicolau/olimp/obm-l.html
=


Re[2]: [obm-l] geometria

2004-09-08 Por tôpico Marcos Paulo

MC = 5BM 


=
De:Marcos Paulo [EMAIL PROTECTED]
Para:[EMAIL PROTECTED]
Assunto:Re: [obm-l] geometria

[X] - área da figura X

(PC/PA)*(QA/QM)*(BM/BC)= 1 (teorema de Menelaus)
BC = 6BM - MC = 6BM - [AMC]=5S/6
[AQP]/[AMC] = (AQ/AM)*(AP/AC)=(3/4)*(2/3)
[AQP] = 0,5*[AMC]=5S/12
[AQP] = 3*[QPM]
[QPM] = 5S/36

[]'s MP




=
De:eritotutor [EMAIL PROTECTED]
Para:obm-l [EMAIL PROTECTED]
Assunto:[obm-l] geometria

Boa noite, 

Gostaria, por favor, de um auxilio na seguinte
questao:

Consideremos ABC um triangulo e  AM e BP são
cevianas 
desse triangulo, sendo M um ponto do segmento
BC
e P  
um ponto do segmento AC.
Essas cevianas se interceptam num ponto Q.
Sabendo que 
a area do triangulo ABC eh S, que AP = 2PC e
que
AQ = 3QM. 
Calcular o valor da area do triangulo PQM em
funçao 
area do triangulo ABC.

Obrigado
 
___
_
__
Acabe com aquelas janelinhas que pulam na sua
tela.
AntiPop-up UOL - É grátis!
http://antipopup.uol.com.br/



===
=
=
Instruções para entrar na lista, sair da lista
e
usar a lista em
http://www.mat.puc-rio.br/~nicolau/olimp/obm-l.
h
tml
===
=
=









=
Instruções para entrar na lista, sair da lista e
usar a lista em
http://www.mat.puc-rio.br/~nicolau/olimp/obm-l.h
tml

=







=
Instruções para entrar na lista, sair da lista e usar a lista em
http://www.mat.puc-rio.br/~nicolau/olimp/obm-l.html
=


Re[2]: [obm-l] geometria

2004-09-09 Por tôpico Marcos Paulo
Ceviana é qualquer reta que passe por um vértice do triângulo. Mediana é somente uma 
ceviana possível de se traçar.
O nome é sim devido  ao Ceva que provou um teorema importante que permite decidir se 
três cevianas traçadas dos três vértices de um triângulo se encontram ou não em um só 
ponto (como é o caso das cevianas mais conhecidas: alturas, medianas e bissetrizes 
internas).

[]'s MP



=
De:[EMAIL PROTECTED]
Para:[EMAIL PROTECTED]
Assunto:Re: [obm-l] geometria

Os termos cevianas e medianas são a mesma
coisa ? Parece que ceviana é uma homenagem a
Ceva (geômetra), não é isso ?



Em uma mensagem de 8/9/2004 20:09:17 Hora padrão
leste da Am. Sul, [EMAIL PROTECTED]
escreveu:




Boa noite, 

Gostaria, por favor, de um auxilio na seguinte
questao:

Consideremos ABC um triangulo e  AM e BP são
cevianas 
desse triangulo, sendo M um ponto do segmento BC
e P  
um ponto do segmento AC.
Essas cevianas se interceptam num ponto Q.
Sabendo que 
a area do triangulo ABC eh S, que AP = 2PC e que
AQ = 3QM. 
Calcular o valor da area do triangulo PQM em
funçao 
area do triangulo ABC.

Obrigado







=
Instruções para entrar na lista, sair da lista e usar a lista em
http://www.mat.puc-rio.br/~nicolau/olimp/obm-l.html
=


Re: [obm-l] Regras aritméticas

2004-09-13 Por tôpico Marcos Paulo
Olá amigos da lista,
me deparei com umas regras aritméticas e
gostaria de saber o porquê delas, por exemplo:
*Multiplicação por nove: tome um número,
exemplo, 355. Pegue o trinta e cinco (centena e
dezena), adiciona um e subtrai de 355. Ou
seja,   355 - 36 = 319. Ao 319, para finalizar,
adicione um algarismo à casa das unidades para
que a soma dos algarismos dê um número múltiplo
de nove, ou seja 3195. Esse é o resultado!!

Isso é o mesmo que 3550 - 360 + 5 = 10*355 - 365 = 9*355

[]'s MP





=
Instruções para entrar na lista, sair da lista e usar a lista em
http://www.mat.puc-rio.br/~nicolau/olimp/obm-l.html
=


Re: [obm-l] Pentágono

2004-09-18 Por tôpico Marcos Paulo
A razão entre os lados do pentágono obtido no passo N para o pentágono obtido no passo 
N-1 é a razão áurea. As áreas dos pentágonos formam um a PG infinita de razão igual ao 
quadrado da razão áurea.

[]'s MP



=
De:Alan Pellejero [EMAIL PROTECTED]
Para:[EMAIL PROTECTED]
Assunto:[obm-l] Pentágono

Olá colegas da lista, não consigo encontrar uma
maneira fácil de resolver esse problema...As
contas dão tantas voltas que acho que estou indo
pelo caminho errado...
 
Tem-se um pentágono de lado a. A partir da
interseção de suas diagonais, forma-se um outro
pentágono. A partir da interseção desse outro
pentágono, pode-se afirmar que:
 
1) Formar-se-á um outro pentágono;
2) Generalizando, se procedermos dessa maneira,
então sempre teremos novos pentágonos;
4) Caso a afirmação acima seja verdadeira, então
se somarmos as infinitas áreas formadas, teremos
o valor da soma Sn = a*[1 + 3^(1/2)]
Justifique.
 
Grato!
Yahoo! Messenger 6.0 - jogos,
emoticons sonoros e muita diversão. Instale






=
Instruções para entrar na lista, sair da lista e usar a lista em
http://www.mat.puc-rio.br/~nicolau/olimp/obm-l.html
=


Re: [obm-l] QUESTCO_MUITO_DIFICIL

2004-10-17 Por tôpico Marcos Paulo
Brunno wrote:
Ola pessoal alguém pode me ajudar neste também 
O valor de cos x + sen x, sabendo que 3.sen x + 4.cos x = 5,
Obrigado
 

Divida toda a expressão por 5 e então vc terá:
0,6senx + 0,8 cos x = 1
Seja cos A = 0,6 e sen A = 0,8 então:
sen x cos A + sen A cos X = 1
sen (x + A) = 1
Daí segue que  x e A são ângulos complementares (somam 0,5 *pi rad) e 
que sen x = cos A = 0,6, assim como cos x = sen A = 0,8
sen x + cos x = 1,4 = 7/5

[]'s MP
=
Instruções para entrar na lista, sair da lista e usar a lista em
http://www.mat.puc-rio.br/~nicolau/olimp/obm-l.html
=


Re: [obm-l] Listagem de premiados da OMERJ

2004-11-17 Por tôpico Marcos Paulo
Muito bacana ver tantos Colégios Municipais sendo premiados. Em
particular com muita alegria eu percebi que a escola que estudei (Escola
Municipal Minas Gerais) vai ser também premiada. É uma pena que na minha
época (1978 até 1986) a escola não participava de competições
Matemáticas. Parabéns a toda a equipe organizadora da OMERJ!
[]'s MP
=
Instruções para entrar na lista, sair da lista e usar a lista em
http://www.mat.puc-rio.br/~nicolau/olimp/obm-l.html
=


Re: [obm-l] Duvidas

2004-11-19 Por tôpico Marcos Paulo
aryqueirozq wrote:
Quantos números reais satisfazem a equação
(x^2 -5x+7)^x+1 =1 ?
 

a) 0b) 1c) 2d) 3 e) 4
 

 

 Me ajudem , nesta equação , só estou achando - 1 como resposta( logo 
uma soluçaõ), mas o gabarito está dando como resposta letra D. Quais 
são as outras soluções?

 

 

Agradeço.
1 elevado a qualquer núemro tb dá 1 .. portanto x^2 - 5x + 7 = 1 tambem 
é solução (x = 2 ou x =3)
=
Instruções para entrar na lista, sair da lista e usar a lista em
http://www.mat.puc-rio.br/~nicolau/olimp/obm-l.html
=


Re: [obm-l] Trapezio e Paralelogramo

2004-12-15 Por tôpico Marcos Paulo
Claudio Buffara wrote:
Questoes de definicao:
1) Um trapezio tem exatamente um par de lados opostos paralelos ou pode ter
ambos os pares de lados opostos paralelos?
2) Se o segundo caso for verdade, posso dizer que um paralelogramo eh um
trapezio isosceles?
[]s,
Claudio.
 

Na minha opinião se vc quiser se expressar direito, ou seja, se vc 
quiser dizer uma coisa e ter certeza de q a mensasgem está clara, um 
trapézio deve ter um único par de lados paralelos.  Claro que os 
paralelogramos têm todas as propriedades dos trapézios mas eu não gosto 
de escrever Se um trapézio é inscritível então ele é isósceles e ser 
refutado com o contra-exemplo de um paralelogramo não retângulo.
Acredito que, desde q vc se mantenha fiel a sua definição, não haverá 
perda nenhuma em considerar paralelogramos paralelogramos e trapézios 
trapézios.

[]'s MP
=
Instruções para entrar na lista, sair da lista e usar a lista em
http://www.mat.puc-rio.br/~nicolau/olimp/obm-l.html
=


Re: [obm-l] Trapezio e Paralelogramo

2004-12-15 Por tôpico Marcos Paulo
Claudio Buffara wrote:
No entanto, considere o caso de um retangulo. Voce pode encara-lo como o
limite de uma sequencia de trapezios isosceles (na sua definicao - um unico
par de lados paralelos) quando o angulo entre os lados nao paralelos tende a
zero. Ou seja, o limite dessa sequencia de trapezios isosceles nao eh um
trapezio isoceles (o que nao eh problema algum, veja bem), apesar de ainda
ser inscritivel.
 

Realmente eu não vejo nenhum problema também, afinal vários números 
irracionais são limites de sequencias de números racionais

Alem disso, o quadrilatero cujos vertices sao:
A = (0,2), B = (1,0), C = (3,2), D = (5,0)
eh ou nao um trapezio?
Em caso afirmativo, o que acontece se fizermos o vertice D se aproximar
arbitrariamente do ponto (4,0)?
Nesse caso, pela sua definicao, ABCD serah trapezio (nao necessariamente
isosceles, claro) para todo D no eixo x, exceto D = (4,0). Nao eh meio
esquisisto?
 

Muito esquisito, como muitas propriedades dos quadriláteros 
entrecruzados (a soma dos ângulos internos - ABC + BCD + CDA + DAB, por 
exemplo, é menor que 360º, a não ser que vc convencione um sentido 
positivo e um negativo para os ângulos) .

No fim, acho que nao vai ter muito jeito. Vamos ter que escolher
arbitrariamente uma definicao, de preferencia aquela que acarretar o menor
numero possivel de emendas nos enunciados de teoremas.
 

Concordo plenamente.
[]'s MP
=
Instruções para entrar na lista, sair da lista e usar a lista em
http://www.mat.puc-rio.br/~nicolau/olimp/obm-l.html
=


Re: [obm-l] questão do livro do Mestre Wagner

2005-01-24 Por tôpico Marcos Paulo
Palmerim Soares wrote:
Ola pessoal,
Alguem poderia me ajudar com a questao 57 do livro I de geometria do 
Mestre Wagner? Seguinte:
O Ponto D eh o pe da bissetriz do angulo reto A do triangulo retangulo 
ABC. Pelo ponto D, traça-se uma perpendicular ao lado BC a qual 
intercepta o lado AC no ponto E. Qual a medida do angulo EBD?
Obrigado,
Palmerim


Yahoo! Acesso Grátis 
http://br.rd.yahoo.com/mail/taglines/*http://br.acesso.yahoo.com/ - 
Internet rápida e grátis. Instale o discador do Yahoo! agora. 
O quadrilátero AEDB é inscritível, já que EAB = BDE = 90 graus. Os 
ângulos DAE e EBD estão inscritos no mesmo arco DE do círculo que 
circunscreve o quadrilátero AEDB, portanto são iguais. A medida de DEA é 
45 graus, já que AD é bissetriz do ângulo reto.
[]'s MP
=
Instruções para entrar na lista, sair da lista e usar a lista em
http://www.mat.puc-rio.br/~nicolau/olimp/obm-l.html
=


Re: [obm-l] dizima

2005-06-22 Por tôpico Marcos Paulo

Brunno Fernandes wrote:


Ola pessoal do grupo poderiam me ajudar?
Dizer quantos algarismos podera ter o período da dizima cuja fração 
geratriz é 25/147
 
eu vi uma regra em que o numero maximo de algarismos da dizima, quando 
o denominador for um numero primo diferente de 2 ou 5, é só pegar o 
numero e subtarir uma unidade, mas 147 não é primo e decompondo em 
fatores primos nao é possivel aplicar essa regra
 
Um abraco

Brunno


OPs falha minha .. o período vai ser 25A somente se eu usar o menor 
valor de n (da resposta anterior) desculpem a falha ;-)


[]'s MP
=
Instruções para entrar na lista, sair da lista e usar a lista em
http://www.mat.puc-rio.br/~nicolau/olimp/obm-l.html
=


Re: [obm-l] dizima

2005-06-22 Por tôpico Marcos Paulo

Brunno Fernandes wrote:


Ola pessoal do grupo poderiam me ajudar?
Dizer quantos algarismos podera ter o período da dizima cuja fração 
geratriz é 25/147
 
eu vi uma regra em que o numero maximo de algarismos da dizima, quando 
o denominador for um numero primo diferente de 2 ou 5, é só pegar o 
numero e subtarir uma unidade, mas 147 não é primo e decompondo em 
fatores primos nao é possivel aplicar essa regra
 
Um abraco

Brunno


O Professor José Paulo Q. Carneiro escreveu um artigo muito interessante 
na RPM 52 sobre dízimas periódicas. NEsse artigo há um método que não 
usa o algoritmo tradicional da divisão e consiste em tentar encontrar 
uma potencia de 10 que deixe resto 1 na divisão por 147.

Seja n tal que 10^n = A*147 + 1, ou seja,
147 = (10^n - 1)/A
1/147 = A*1/(10^n - 1) = A* [(1/10^n) /(1-1/10^n)] = A/10^n *[1 + 1/10^n 
+ 1/10^2n + ...]

25/147 = 25A/10^n *[1 + 1/10^n + ...]

O número 25A será seu período.

No artigo ainda há uma explicação de como ter certeza de que haverá uma 
potência de 10 que deixe resto 1 na divisão por 147 (que é primo com 
10). A certeza vem de um dos teoremas de Euler que garante que 
10^[phi(147)] deixa resto 1 na divisão por 147, onde phi(147 é o número 
de inteiros positivos menores que 147 e primos com 147. Phi(147) = 
147(1-1/3)(1-1/7) = 84. Isso me permite reduzir a procura das potencias 
de 10, bastando testar apenas os expoentes que são divisores de 84.
Não digo que é o melhor método ou que é o mais apropriado mas é uma 
alternativa interessante e o artigo é muito legal.. vale a pena ler.


[]'s MP
=
Instruções para entrar na lista, sair da lista e usar a lista em
http://www.mat.puc-rio.br/~nicolau/olimp/obm-l.html
=


Re: [obm-l] Re: [obm-l] Dúvida denovo sobre Analise Combinatória

2005-07-18 Por tôpico Marcos Paulo

Carlos Gomes wrote:


A maneira coreta é C_9,3 . C_6,3.C_3,3 = 1680
Cgomes
- Original Message - From: Gabriel Bastos Gomes 
[EMAIL PROTECTED]

To: obm-l@mat.puc-rio.br
Sent: Sunday, July 17, 2005 6:06 PM
Subject: [obm-l] Dúvida denovo sobre Analise Combinatória


Ae pessoal... Eu estou me matando pra recuperar essa matéria, porém 
surgiu
uma nova dúvida. Eu não queria ser tão incoveniente... Se puderem 
matar essa

dúvida, agradeço...

(EAESP-FGV) Nove pessoas param para pernoitar num motel. Existem 3 
quartos

com 3 lugares cada. O número de formas com que essas pessoas podem se
distribuir entre os quartos é:

a) 84
b) 128
c) 840
d) 1680
e) 3200

Minha resolução deu C_9,3 * C_3,3 = 84 ... So que eu estou muito inseguro
nessa matéria... Se alguém puder confirmar...

Abraços,
Gabriel

_
MSN Messenger: converse online com seus amigos .
http://messenger.msn.com.br

=
Instruções para entrar na lista, sair da lista e usar a lista em
http://www.mat.puc-rio.br/~nicolau/olimp/obm-l.html
=

Em combinatória normalmente há várias maneiras corretas . A maneira que 
o carlos apontou como correta é apenas uma das soluções corretas que 
consiste em 3 decisões independentes: 1) Escolher 3 dos 9 para preencher 
o primeiro quarto (C9,3), 2) escolher 3 dos 6 restantes para preencher o 
segundo quarto(C6,3) e 3) Colocar o restante das pessoas no terceiro 
quarto(1). Como as decisões são independentes pode-se usar o principio 
fundamental da contagem obtendo a resposta indicada.
Outra maneira de pensar o mesmo problema: Suponha que cada quarto tenha 
3 camas, então o problema consiste em colocar 9 pessoas em 9 camas, o 
que pode ser feito de 9! maneiras. Acontece que a ordem das camas em 
cada quarto não altera a disposição das pessoas nos quartos, então cada 
uma das possibilidades foi contada 216 vezes e a resposta passa a ser 
9!/216 =1680.
Assim como essa segunda maneira de ver o problema (talvez menos natural 
do que a primeira) há diversas outras maneiras de se chegar ao resultado 
correto (até mesmo fazendo a listagem de todas as disposições 
possíveis). Se vc está estudando combinatória é interessante que vc 
mantenha a cabeça aberta para várias maneiras de resolver o mesmo 
problema. Assim vc terá mais opções de estratégias em outros exercícios.


[]'s MP
=
Instruções para entrar na lista, sair da lista e usar a lista em
http://www.mat.puc-rio.br/~nicolau/olimp/obm-l.html
=


Re: [obm-l] Re: [obm-l] dúvida conceitual

2005-07-18 Por tôpico Marcos Paulo

Domingos Jr. wrote:


Carlos Gomes wrote:

Claro que não, pois os vetores de uma base do R^2 tem duas 
coordenadas enquanto que os vetores do r^3 tem 3 coordenadas!




Podemos pensar um pouquinho fora da caixa...
Dois vetores LI no R^3 determinam um (hiper-)plano que é isomorfo ao R^2.
Acho que esse tipo de resposta é mais informativa do que um 'claro que 
não'.



=
Instruções para entrar na lista, sair da lista e usar a lista em
http://www.mat.puc-rio.br/~nicolau/olimp/obm-l.html
=


Os vetores (1,0,0)  e (1, 0, 1) são LI e não geram um hiperplano 
isomorfo ao R²


[]'s MP
=
Instruções para entrar na lista, sair da lista e usar a lista em
http://www.mat.puc-rio.br/~nicolau/olimp/obm-l.html
=


Re: [obm-l] Re: [obm-l] dúvida conceitual

2005-07-18 Por tôpico Marcos Paulo

Domingos Jr. wrote:


Carlos Gomes wrote:

Claro que não, pois os vetores de uma base do R^2 tem duas 
coordenadas enquanto que os vetores do r^3 tem 3 coordenadas!




Podemos pensar um pouquinho fora da caixa...
Dois vetores LI no R^3 determinam um (hiper-)plano que é isomorfo ao R^2.
Acho que esse tipo de resposta é mais informativa do que um 'claro que 
não'.



=
Instruções para entrar na lista, sair da lista e usar a lista em
http://www.mat.puc-rio.br/~nicolau/olimp/obm-l.html
=


OPs falei besteira na ultima mensagem desconsiderem por favor. (eu 
estava pensando só nos planos paralelos ao XY no R³)


[]'s MP
=
Instruções para entrar na lista, sair da lista e usar a lista em
http://www.mat.puc-rio.br/~nicolau/olimp/obm-l.html
=


Re: [obm-l] Usando o Maple

2005-07-23 Por tôpico Marcos Paulo

Carlos Victor wrote:


Olá  pessoal ,

Gostaria  de saber  como calcular  diretamente  usando o  Maplea 
seguinte  soma  :


 *(5+2*sqrt(13))^(1/3) + (5-2*sqrt(13))^(1/3)

*Agradeço  desde já  qualquer  ajuda


[]´sCarlos  Victor



Tente substituir o 13 por 13.0

[]'s MP
=
Instruções para entrar na lista, sair da lista e usar a lista em
http://www.mat.puc-rio.br/~nicolau/olimp/obm-l.html
=


Re: [obm-l] Usando o Maple

2005-07-24 Por tôpico Marcos Paulo

Carlos Victor wrote:



Olá  Marcos,

Esta  substituição   não  resolveu ;  caso  exista  outra me informe , 
ok ?


[]´s  Carlos Victor

At 01:36 24/7/2005, Marcos Paulo wrote:


Carlos Victor wrote:


Olá  pessoal ,

Gostaria  de saber  como calcular  diretamente  usando o  Maplea 
seguinte  soma  :


 *(5+2*sqrt(13))^(1/3) + (5-2*sqrt(13))^(1/3)

*Agradeço  desde já  qualquer  ajuda


[]´sCarlos  Victor


Tente substituir o 13 por 13.0

[]'s MP
= 


Instruções para entrar na lista, sair da lista e usar a lista em
http://www.mat.puc-rio.br/~nicolau/olimp/obm-l.html
= 






=
Instruções para entrar na lista, sair da lista e usar a lista em
http://www.mat.puc-rio.br/~nicolau/olimp/obm-l.html
=


O problema está com a raíz cpubica da segunda parcela porque (por alguma 
razão) o mapple te fornece uma das raízes complexas. Dá pra contornar 
(porque a raiz cúbica é uma função ímpar) calcule:


 (5+2*sqrt(13.0))^(1/3) - (-5+2*sqrt(13.0))^(1/3);

[]'s MP
=
Instruções para entrar na lista, sair da lista e usar a lista em
http://www.mat.puc-rio.br/~nicolau/olimp/obm-l.html
=


Re: [obm-l] Problema simples: chegar em 24, com 1, 3, 4 e 6

2005-08-15 Por tôpico Marcos Paulo

Susanna wrote:


Olá! acabei de entrar na lista da OBM e, ao ollar o arquivo de
e-mails, percebi que os problemas sugeridos, em sua maioria, requeriam
um certo conhecimento previo de matemática. Porém os problemas mais
intrigantes são os mais simples e ao mesmo tempo difíceis de encontrar
uma solução mas uma vez encontrada, parece tão óbvia.

esse problema foi proposto no ônibus de um congresso internacional de
matemática que foi realizado na França em maio desse ano. intrigou os
matemáticos a ponto de uns dizerem que perderam o sono.

Eis o problema:
usando todos e sem repetir nenhum dos numeros: 1, 3, 4, e 6 e usando
qualquer das quatro operações básicas (+ - x / ) chegar ao número 24.

obs.: Só vale as 4 operações básicas. nada de exponencial e nada de
juntar dois números (fazer 13 com 1 e 3).

boa sorte,
susanna

=
Instruções para entrar na lista, sair da lista e usar a lista em
http://www.mat.puc-rio.br/~nicolau/olimp/obm-l.html
=


 


2+3-5 + 1*4*6
[]'s MP
=
Instruções para entrar na lista, sair da lista e usar a lista em
http://www.mat.puc-rio.br/~nicolau/olimp/obm-l.html
=


Re: [obm-l] setores circulares

2005-09-19 Por tôpico Marcos Paulo

Renato G Bettiol wrote:


Carissimos,
hoje me entreti bastante na resolução do seguinte problema, 
aparentemente de um vestibular da UFMG:
 
*Dentre setores circulares de mesmo perímetro, determinar aquele de 
maior área.*
 
Vale a pena tentar, fazer recorrendo ao cálculo do valor máximo da 
função quadrática que relaciona o comprimento
do arco do setor em questão e sua área. Ao fim das contas dará que o 
ângulo central deve ser igual a 2rad.

Quem se interessar na resolução, mande um e-mail,
 
Abraços
 
Renato


Seja /x/ o lado do círculo e /l/ o comprimento do arco. Tomando como 
unidade o perímetro do setor tem-se 2/x+l = /1, ou seja /l /= 1-2/x/. Se 
/a/ é o ângulo central, então /a/ = (1-2/x)///x/. A área do setor vale 
/S(x)/ = /x²*/(1-2/x/)/2/x/ =/ x/(1-2/x/)/2. /S(x) /será máxima para 
/x/=1/4, ou seja quando o ângulo /a = 2 rad/.


Uma outra maneira como /2x + l /é constante, o produto de /2x/ e /l/ (do 
qual a área é um quarto) será máximo quando /2x = l =/ 1/2, ou seja x = 
1/4 e assim continua...


[]'s MP
=
Instruções para entrar na lista, sair da lista e usar a lista em
http://www.mat.puc-rio.br/~nicolau/olimp/obm-l.html
=


Re: [obm-l] USO INTELIGENTE DA CALCULADORA!

2005-11-17 Por tôpico Marcos Paulo

Jorge Luis Rodrigues e Silva Luis wrote:

Dispondo de uma calculadora de 8 dígitos no visor, como obter todos os 
algarismos do número 2^64?


Dica: Um caminho promissor é decompor 2^32 em três parcelas 
convenientemente escolhidas e, em seguida, utilizar a fórmula (a + b + 
c)^2



De posse de uma calculadora de alta precisão, o aluno efetua o produto 
de 95 noves por 95 cincos e obtém um número cuja soma dos algarismos é 
igual a...



Na minha calculadora, uma das teclas de 1 a 9 está com defeito: ao 
pressioná-la aparece na tela um dígito entre 1 e 9 que não é o 
correspondente. Quando tentei escrever o número 987654321, apareceu na 
tela um número divisível por 11 e que deixa resto 3 ao ser dividido 
por 9. Qual é a tecla defeituosa? Qual é o número que apareceu na tela?


Abraços!

_
MSN Messenger: converse online com seus amigos .  
http://messenger.msn.com.br


=
Instruções para entrar na lista, sair da lista e usar a lista em
http://www.mat.puc-rio.br/~nicolau/olimp/obm-l.html
=


Os problemas são até bacanas mas e a calculadora nisso tudo? A mensagem 
junto com o subject sugere que o uso inteligente da calculadora é não 
usa-la (o que eu discordo). O que aparece na mensagem são problemas 
contextualizados em torno de calculadoras e naõ o uso inteligente delas 
.. Acho qeu o problema dos 95 noves e 95 cincos eh olimpico (estadual do 
rj ou brasileira acho) e o enunciado na prova não falava nada de 
calculadora..


[]'s MP
=
Instruções para entrar na lista, sair da lista e usar a lista em
http://www.mat.puc-rio.br/~nicolau/olimp/obm-l.html
=


[obm-l] Data do resultado OBM

2005-12-19 Por tôpico Marcos Paulo

OI,

existe alguma previsão de quando sai o resultado da OBM?

[]'s MP
=
Instruções para entrar na lista, sair da lista e usar a lista em
http://www.mat.puc-rio.br/~nicolau/olimp/obm-l.html
=